00 Main
00 Main
00 Main
c May, 2013
Contents
1 Preliminaries 1
1.1 Numbers and Equality . . . . . . . . . . . . . . . . . . . . . . . . . . . . . . 1
1.2 Variables . . . . . . . . . . . . . . . . . . . . . . . . . . . . . . . . . . . . . . 3
1.3 Grouping Symbols . . . . . . . . . . . . . . . . . . . . . . . . . . . . . . . . 5
1.4 Order of Operations . . . . . . . . . . . . . . . . . . . . . . . . . . . . . . . 6
1.5 Properties of Operations . . . . . . . . . . . . . . . . . . . . . . . . . . . . . 8
1.6 Sets . . . . . . . . . . . . . . . . . . . . . . . . . . . . . . . . . . . . . . . . 10
2 Fractions 13
2.1 The concept of a fraction . . . . . . . . . . . . . . . . . . . . . . . . . . . . . 13
2.2 Fraction Equivalence . . . . . . . . . . . . . . . . . . . . . . . . . . . . . . . 18
2.3 Fraction Simplification . . . . . . . . . . . . . . . . . . . . . . . . . . . . . . 23
2.4 Operations on Fractions:
Addition and Subtraction . . . . . . . . . . . . . . . . . . . . . . . . . . . . 26
2.5 Operations on Fractions:
Multiplication and Division . . . . . . . . . . . . . . . . . . . . . . . . . . . 34
2.6 Applications of Fractions . . . . . . . . . . . . . . . . . . . . . . . . . . . . . 45
2.7 Rational Expressions . . . . . . . . . . . . . . . . . . . . . . . . . . . . . . . 51
3 Exponents 59
3.1 Introduction to Exponents . . . . . . . . . . . . . . . . . . . . . . . . . . . . 59
3.2 Properties of Exponents . . . . . . . . . . . . . . . . . . . . . . . . . . . . . 65
3.3 Integer Exponents . . . . . . . . . . . . . . . . . . . . . . . . . . . . . . . . . 76
3.4 Rational Exponents . . . . . . . . . . . . . . . . . . . . . . . . . . . . . . . . 82
3.5 Radical Expressions . . . . . . . . . . . . . . . . . . . . . . . . . . . . . . . . 88
3.6 Scientific Notation . . . . . . . . . . . . . . . . . . . . . . . . . . . . . . . . 92
4 Distributive Laws 97
4.1 The Distributive Law . . . . . . . . . . . . . . . . . . . . . . . . . . . . . . . 98
4.2 Generalizations of the Distributive Law . . . . . . . . . . . . . . . . . . . . . 102
4.3 Multiplication of Polynomial Expressions . . . . . . . . . . . . . . . . . . . . 107
i
ii
5 Lines 121
5.1 Linear equations . . . . . . . . . . . . . . . . . . . . . . . . . . . . . . . . . 121
5.2 Linear inequalities . . . . . . . . . . . . . . . . . . . . . . . . . . . . . . . . 128
5.3 The Rectangular Coordinate System . . . . . . . . . . . . . . . . . . . . . . 137
5.4 The Slope of a Line . . . . . . . . . . . . . . . . . . . . . . . . . . . . . . . . 143
5.5 Equations of Lines . . . . . . . . . . . . . . . . . . . . . . . . . . . . . . . . 153
5.6 Graphs of lines . . . . . . . . . . . . . . . . . . . . . . . . . . . . . . . . . . 163
6 Functions 169
6.1 Definitions . . . . . . . . . . . . . . . . . . . . . . . . . . . . . . . . . . . . . 169
6.2 Notation . . . . . . . . . . . . . . . . . . . . . . . . . . . . . . . . . . . . . . 174
6.3 Piecewise-Defined Functions . . . . . . . . . . . . . . . . . . . . . . . . . . . 182
6.4 Operations on Functions . . . . . . . . . . . . . . . . . . . . . . . . . . . . . 186
This Intermediate Algebra text provides the basic knowledge and skills necessary for success
in college level mathematics courses. Our emphasis is on the study of structure and form,
the relationships among various parts of algebra, the development of analytical reasoning,
and the use of algebra as the language of mathematics. We place emphasis on understanding
rather than on rote manipulation of symbols. We are striving for a mastery of a few basic
principles underlying much of algebra; we believe that such mastery is the best preparation
for grasping mathematics.
These materials do not approach topics in isolation, but integrate them, progressing from
more basic ideas to more advanced ideas. Students learn best when they are actively involved
with the mathematics they are studying, whether it be through explorations with technology,
discovery activities, writing assignments, or extensive exercise sets. Therefore, students who
practice communicating about mathematics in a variety of ways develop a more complete
grasp of the concepts involved.
iii
iv
Preface to the Student
In order to succeed in this course, you need to know our ultimate goals so that you can focus
on them. Our primary goal is for you to acquire an understanding of some fundamental
principles of algebra. Memorizing “the rules” is not what we’re after. When you have a
good grasp of the principles, the rules become natural and do not require rote memorization.
Secondly, we will be studying algebra itself; many of the concepts we discuss will be
familiar, but we will discuss them in much greater depth. It is extremely important that
you not fall into the trap of believing that you already know all about algebra. Because the
topics are familiar, many students fall back on old (and bad) habits, and never learn algebra
at the level they are capable of. Do not make this mistake! Please believe that you will
not do well this semester unless you can clear your mind and start at the beginning with us.
We have divided the book into six units. The first unit discusses some common conven-
tions, symbols, and terms that you should already be familiar with. The second unit is a
detailed exploration of fractions; in particular, we examine why we treat them the way we
do. The third unit is about what exponents are and how they are used. The fourth unit
is about the distributive law. This law is one of the fundamental properties of our number
system; understanding it will help you immensely in your algebra studies. The fifth unit is
about lines, and the sixth unit is an introduction to functions. We approach all of the units
in a fairly informal way, but we do make frequent use of symbols.
The class will not be easy. You will need to spend about three (3) hours outside of class
for every hour in class; that means about 8 hours per week at a minimum. You may need
more time; take that time! This is why college is a full-time job: a student taking 12 hours
should be studying 24 to 36 hours outside of class, for a total of 36 to 48 hours.
If you finish your homework early, go over your notes, read the book, make up more
problems for yourself. When you read the book, read it actively, not passively. Have a
blank piece of paper, and work out all of the details for yourself even if they are in the book
already. Cover them up and work through them again. We know that this is a lot of work,
but if you are engaged at this level, you will do well. Just remember that a math text is
not a novel; to read it properly, you must be actively involved. Mathematics is dense with
symbols; each one carries a lot of meaning. A great deal of care is required to extract that
meaning, but you can do it if you take the time. Commit to it now!
v
vi
Chapter 1
Preliminaries
The set of rational numbers is the set of ratios of integers (with nonzero denominators);
that is, the set of fractions. Numbers such as
2 17 39 −12
, , , −6,
3 −5 13 −34
are all rational numbers. There are several ways to represent any given rational number.
Notice that the rational numbers include the integers. For example, −6 is a rational
number since −6 = −6 1
. Also note that 02 is not a rational number.
1
Example 1.1.1. The fraction 4
is the same as 0.25, and can also be written as 82 , −4
−1
, and 4
16
,
among other ways.
The set of real numbers includes the set of all rational numbers and much more. Real
numbers can be thought of as numbers that correspond to distances or lengths and the √
3
negatives of those numbers. For example, one can travel 10 of a mile, and one can travel 2
√
miles. You may see a proof in a later course that 2 is not a rational number: it cannot be
written as a ratio of integers. However, it does correspond to a distance: it is the length of
the diagonal of a square that is 1 unit long on each side.
Here are three important properties of equality that we will make frequent use of.
1
2
1. Equality is reflexive: every real number is equal to itself. That is, if a is a real number,
then a = a.
2. Equality is symmetric: if a = b, then b = a.
3. Equality is transitive: if a = b and b = c, then a = c.
Example 1.1.2. If we know that 0 = x − 4, we may rewrite this as x − 4 = 0 because equality
is symmetric.
1.2 Variables
In mathematics, we have a frequent need to refer to an unknown number. Consider the
following situation.
Example 1.2.1. Timothy has three times as many eggs as Sarah. If Timothy has 8 more eggs
than Sarah, how many eggs does each have?
We can answer this question as follows: 3 times the number of eggs Sarah has is the
number of eggs Timothy has, and so is 8 more than the number of eggs Sarah has. Thus
3×(the number of eggs Sarah has)=(the number of eggs Timothy has)
and
(the number of eggs Timothy has)=(the number of eggs Sarah has) +8.
Using the transitive property of equality (above), we see that
3×(the number of eggs Sarah has)=(the number of eggs Sarah has)+8.
If we subtract the number of eggs Sarah has from each side of the equation, we get
2×(the number of eggs Sarah has)=8,
so Sarah must have 4 eggs. Therefore, Timothy has 12 eggs.
You may have noticed that writing “the number of eggs Sarah has” and “the number of
eggs Timothy has” was rather awkward and unwieldy. To streamline the process, we choose
a variable to represent each quantity. Let us choose s to represent the number of eggs
Sarah has and t to represent the number of eggs Timothy has. The choices of names for our
variables are completely arbitrary, so we picked names that would help us remember what
each variable stands for: s for Sarah and t for Timothy. Now our equations above look like
3s = t and t = s + 8, so that 3s = s + 8.
Now we can subtract s from both sides to get 2s = 8 and s = 4. That is, Sarah has 4 eggs
and Timothy must have 3 · 4 = 12 eggs.
Using variables to represent the quantities made the equations much more compact. How-
ever, you must be careful to remember what each variable represents, too. If you introduce
a new variable into a problem, you must specify carefully what quantity it is representing.
There is another inherent difficulty in introducing variables: it represents a step toward
a more abstract way of thinking, which can be difficult at first. However, the power you
gain from it is well worth the effort; we will see throughout this course that variables are
extremely powerful and useful.
Example 1.2.2. The Thompsons needed to have sand delivered for a construction project at
their home. The cost of the sand was $8 per cubic yard plus a $10 delivery charge. Express
the total cost in terms of the number of cubic yards of sand they ordered.
Solution: We could express this as
8×(the number of cubic yards of sand the Thompsons had delivered)+10,
4
but that is very awkward. Instead, let’s let s represent the number of cubic yards of sand
they had delivered. Then the cost of getting the sand was
8s + 10.
11. It takes Thadd 15 minutes to set up his equipment every day. Once it is set up, Thadd
can make a designer pen in about 45 minutes. How many pens can Thadd make if he
works for 7 hours?
Example 1.3.2. In the expression 9 + 45 + 12, the sum is 66, and each of 9, 45, and 12 is a
term.
Example 1.3.3. In the expression 5 − 9, the terms are 5 and −9 since 5 − 9 = 5 + (−9). Thus,
subtraction is a form of addition. We will make frequent use of this idea without necessarily
referring to it explicitly, so make sure you understand it.
Example 1.3.4. In the product x(x + 3)(y − 4), x is a factor, x + 3 is a factor, and y − 4 is a
factor. Within the factor x + 3, x is a term and 3 is a term. Within the factor y − 4, y is a
term and −4 is a term since y − 4 = y + (−4).
The parentheses in the previous example are used as grouping symbols. We need
grouping symbols to indicate the order of operations; that is, the order in which we want
to perform the operations we are given. In general, operations within grouping symbols must
be performed before what is grouped can be used in another computation.
Example 1.3.5. (4 + 3) · 9 = 7 · 9. The sum 4 + 3 needs to be computed before we can multiply
by 9 because the sum 4 + 3 is grouped within the parentheses. The parentheses indicate that
what is inside them should be treated as a single number.
Parentheses are not the only grouping symbols; we will encounter many more in our
explorations. Some common grouping symbols are
[ ] and { } ;
they are used in the same way as parentheses. Usually, they are used to make a complicated
expression easier to read.
6
Example 1.3.6. Compute 12 − [4 − (5 − 8)]. The square brackets [ and ] indicate that we
must compute 4 − (5 − 8) before we can subtract that from 12. The parentheses tell us that
in order to compute 4 − (5 − 8), we must first compute 5 − 8 = −3. Thus, we have
12 − [4 − (5 − 8)] = 12 − [4 − (−3)]
= 12 − 7
=5
Notice that the innermost parentheses dictated the first computation. Also notice that
12 − [4 − (5 − 8)] is easier to read than 12 − (4 − (5 − 8)). The different grouping symbols
help to guide our eyes.
Express each mathematical expression verbally using the terms “sum” and “product” as
appropriate.
7. 4 · 5 9. 3 · 9 + 15 11. 5 + 9(8 + 2)
Compute as indicated.
4+7·8
could be ambiguous. Do we add the 4 and the 7 first, and then multiply by 8, or do we
multiply 7 and 8 first and then add the result to 4? The first way gives us 88 and the second
way gives us 60, so it will make a difference.
To avoid this ambiguity, we need to all agree on which operations come first. Mathe-
maticians have adopted a convention: multiplication and division receive a higher priority
than addition and subtraction; that is, they are performed first, as though the product had
been written inside of grouping symbols.
Example 1.4.1. The second interpretation above is the correct one:
4 + 7 · 8 = 4 + (7 · 8)
= 4 + 56
= 60.
When there are several multiplications and/or divisions, they are performed from left to
right; likewise, when there are several additions and/or subtractions, they are also performed
from left to right.
Example 1.4.2.
6 · 12 ÷ 9 · 4 = [(6 · 12) ÷ 9] · 4
= [72 ÷ 9] · 4
= 8 · 4 = 32.
Example 1.4.3.
Example 1.4.4.
7 + 5 · 8 ÷ 2 − 14 = {7 + [(5 · 8) ÷ 2]} − 14
= {7 + [40 ÷ 2]} − 14
= {7 + 20} − 14
= 27 − 14
= 13.
Notice that even though the multiplication and division were in the middle of the expres-
sion, they were performed first.
1. 41 − 18 + 6 7. (28 ÷ 4) + 3
3. 17 − 8 − 5 9. 3 · 3 · 3 · 3
4. 5 · 4 + 5 · 2 10. 4 ÷ (8 + 12) · 80
5. 28 ÷ 4 + 3 11. 8 − {5 − [3 − (6 − 4)]}
6. 28 ÷ (4 + 3) 12. (6 − 3) ÷ 2 + 1
13. 4 · 3 + 1 17. 4 + 12 ÷ (2 · 3) − 3 − 4.
14. 4 + 5 · 3 + 2 18. 8 + 2 + 5 − 3
15. 5 − 3 − 4 19. 4 · 6 ÷ 2 · 11
16. 6 · 7 ÷ 2 20. 2 − 15 ÷ 3 + 2
a + (−a) = (−a) + a = 0.
9. Every real number except 0 has a multiplicative inverse: if a is a real number, then
1 1
a· = · a = 1.
a a
Example 1.5.1. Computation yields that (23.5) · 9 = 211.5 and 9 · (23.5) = 211.5 This
illustrates the commutativity of multiplication.
Example 1.5.3. Since 0 is the additive identity, we have that 4127 + 0 = 4127 and 0 + 4127 =
4127.
Example 1.5.4. Since 8 · (0.125) = (0.125) · 8 = 1, we see that 8 is the multiplicative inverse
of 0.125 and 0.125 is the multiplicative inverse of 8.
2. (6 · 5) · 2 = 6 · (5 · 2) 8. (x + y) · (a + b) = (a + b) · (x + y)
13. A customer in a back aisle of your store has seven items worth $1.52 each. She tells you
that her partner will soon be arriving with three more of the same item. What field
property allows you to quickly compute the total value of the items, 7 · 1.52 + 3 · 1.52?
14. Identify which field property is used in each step below.
(x + 2)(x + 3) = [(x + 2) · x] + [(x + 2) · 3]
= [x · (x + 2)] + [3(x + 2)]
= [x · x + x · 2] + [3 · x + 3 · 2]
= [x2 + 2x] + [3x + 6]
= [(x2 + 2x) + 3x] + 6
= [x2 + (2x + 3x)] + 6
= [x2 + (2 + 3)x] + 6
= [x2 + 5x] + 6 2+3=5
2
= x + 5x + 6
11
15. In algebra, we often have a need to “combine like terms.” For example, 7x + 5x = 12x.
What property of operations allows us to combine like terms? Show that 7x+5x = 12x
using the properties of operations.
1.6 Sets
We often have collections of objects that we need to describe in some compact way. In this
section, we give a brief introduction to the idea of a set.
A set is a collection of objects described unambiguously. Thus, “the collection of people
over 6 feet tall” is a set, but “the collection of tall people” is not a set. Most of the time,
we will be interested in sets of numbers, but we will not restrict ourselves to them.
We have two main ways of writing sets. The first is roster notation.
Example 1.6.1. The set of natural numbers less than 7 is written in roster notation as
{1, 2, 3, 4, 5, 6}. The “curly braces” { and } indicate where the set begins and ends.
Example 1.6.2. The set of numbers satisfying the equation x2 = 4 is {2, −2}.
Example 1.6.3. The set of natural numbers is {1, 2, 3, 4, 5, 6, . . .}. The dots . . . indicate that
the established pattern is to be continued.
Our second notation is called set-builder notation; we use it when sets are too big or
too complicated for us to actually list all of the elements or even establish a pattern.
Example 1.6.4. Consider the set consisting of all real numbers except zero. We can’t list them
all, and we can’t even describe a nice pattern. However, we have a convenient shorthand
way to describe this set, too. We write
{x|x 6= 0},
where again the curly braces { and } indicate the beginning and ending of the set. Here are
what the symbols all mean.
{ x | x 6= 0 }
The set of all x such that x is not equal to 0 .
The vertical bar comes right before the condition that determines whether or not x gets
to be in the set. That is, the condition right after the vertical bar is the “entrance exam” x
must pass to get into the set.
Example 1.6.5. {x|x ≥ 0} is the set of all real numbers that are zero or greater.
12
Example 1.6.6. {x|x 6= 1, 2, 3, 4} is the set of all real numbers except 1, 2, 3, and 4.
Chapter 2
Fractions
Recall from Chapter 1 that the rational numbers are those numbers that can be written
as a ratio of integers (with nonzero denominators); they are the fractions. We will develop
our usual rules for adding, subtracting, multiplying, and dividing fractions by looking at the
models that illustrate these concepts.
We say that each piece is one eighth of the pie. The eight equal pieces together comprise
the whole pie; that is, if we take eight eighths, we have one whole. We represent these
quantities with fractions, which we represent with symbols of the form ab , where a and b
are integers and b 6= 0.
The fraction 18 carries two pieces of information. The 8 in the denominator indicates the
number of equal pieces the whole has been cut into. The numerator indicates the number
of those pieces we have. Thus, the fraction 83 indicates that the pie has been cut into 8 equal
pieces, and we have 3 of those. In Figure 2.2, the shaded portion is the portion we have.
13
14
1 2 3 4 5 6 7 8
=1
8 8 8 8 8 8 8 8
Figure 2.2: Eighths
This is the basic premise of fractions. Fractions are another way of considering division
since we begin by dividing a unit into equal pieces. In fact, you may have noticed that the
division symbol, ÷, looks like a fraction bar with little dots for placeholders in the numerator
and denominator.
There are many models for fractions; in the examples below, we consider some of these.
Example 2.1.1. The rectangle below has been divided into 10 equal pieces, 4 of which are
4
shaded. The shaded portion therefore represents 10 .
4
Figure 2.3:
10
2
Figure 2.4:
5
Example 2.1.3. Some fractions consist of more than one whole unit. The figure below illus-
17
trates the fraction 12 .
15
17
Figure 2.5:
12
5 5
This can also be written as 1 + 12 , since it is a whole unit plus 12 more. Notice that the
unit is one strip, and it has been divided into 12 equal pieces. Since we have 17 of those
pieces shaded, the fraction represented is 1712
.
Sometimes a fraction such as 17
12
will appear as a mixed number:
17 5
=1 .
12 12
When a fraction is written this way, it means
5
1+ .
12
Because the mixed number notation looks like multiplication, our use of mixed numbers in
this text will be very limited.
This model illustrates a strip model of fractions.
Example 2.1.5. The famous number π ≈ 3.14159 is not a rational number; it cannot be
written as ab with a and b integers. The digits to the right of the decimal neither terminate
nor repeat; proving this requires techniques that are beyond the scope of this course.
16
3
Example 2.1.6. The symbol does not represent a rational number since the denominator
0
is 0.
1
Example 2.1.7. For the symbol to represent a rational number, we must have x 6= 0.
x
12
Example 2.1.8. For the symbol to represent a rational number, we must have x 6= −3.
x+3
Example 2.1.9. If a is any integer, then the fraction a1 is just equal to a. Why is this? The
1 in the denominator indicates that we are taking a unit and dividing into 1 piece; that is,
we aren’t dividing it at all. Since we then take a of those unit-sized pieces, we have a total
of a units!
Thus,
3 −12 0
= 3, = −12, and = 0.
1 1 1
Remember that fractions refer to division. This means that 54 is the same as 4 ÷ 5. We
can use this division idea to find decimal equivalents of fractions.
4
Example 2.1.11. Since 4 ÷ 5 = 0.8, we see that = 0.8.
5
7. 0.25 9. 1.5
8. 0.4 10. 0.6
11.
14.
12.
1 unit
15.
13. 16.
3 6
4 8
3 6
Figure 2.6: =
4 8
The shaded regions show that 68 = 34 . Let’s think about what the symbols all mean. The
symbol 86 means that we have divided a unit into 8 equal pieces and taken 6. Notice that if
we gather the 8 equal pieces into groups of 2 pieces each, we will have exactly 4 groups, all
of equal size. Of those, we have a total of 3 groups of 2 pieces each that are shaded. That
is exactly what 34 means: we have divided the unit into 4 equal pieces and taken 3!
This concept is known as fraction equivalence. Intuitively, two fractions are equivalent
if they represent the same quantity.
If two fractions have the same denominator, then the fractions are equivalent if and only
if they have the same numerator.
4 7
Example 2.2.1. The fractions 15 and 15 have the same denominator; therefore, they are not
equivalent because their numerators are not the same.
2
Figure 2.7:
4
Without changing the portion shaded, we can subdivide those 4 pieces into 3 pieces each,
so we end up with a total of 4 · 3 = 12 pieces. Originally 2 pieces were shaded, and we see
6
that now 2 · 3 = 6 pieces are shaded, so we have 12 shaded. Thus, 24 and 126
are equivalent
fractions.
6 2
Figure 2.8: =
12 4
1 1·5 5
= = ,
3 3·5 15
as the figure illustrates.
1 5
Figure 2.9: =
3 15
20
4 2
Example 2.2.4. The fractions 6
and 3
are equivalent since both of them are equal to 32 . Also,
we may write 64 as 2·2
3·2
.
7 42
Example 2.2.5. The fractions 3
and 18
are equivalent since
7 7·6 42
= = .
3 3·6 18
Example 2.2.6. This idea works even when the numbers are not known; for example,
x 4x
is equivalent to .
x+1 4(x + 1)
Note that we must have the implicit assumption that x 6= −1 for the fractions in this example
to be defined.
8
Example 2.2.7. Show that 12 and 10
15
are equivalent.
Solution: We have seen that it is easy to compare fractions with the same denominator.
8
Our strategy, therefore, will be to find a fraction equivalent to 12 and another fraction
10
equivalent to 15 , where both new fractions have the same denominator. One way to get a
common denominator is to multiply each denominator by the other: since 12 · 15 = 15 · 12,
the new denominators will be equal.
8 8 · 15 120 10 10 · 12 120
= = and = = .
12 12 · 15 12 · 15 15 15 · 12 12 · 15
Now these two fractions have the same denominator (15 · 12) and both have a numerator
equal to 120. Therefore, they are equivalent.
We can also use a rectangular array model to see that these are equivalent. Below are
8 10
models for 12 and 15 .
8 10
Figure 2.10: and
12 15
21
8
We will subdivide the model for 12 into 15 equal pieces by using horizontal lines. We will
10
also subdivide the model for 15 into 12 equal pieces by using vertical lines. Remember, this
doesn’t change at all the amount of the diagram that is shaded.
8 10
Figure 2.11: and subdivided
12 15
Both rectangles are the same size, and both are divided into 15 · 12 = 12 · 15 = 180 equal
8
pieces. In the diagram for 12 , we have 8 · 15 = 120 pieces shaded. In the diagram for 10
15
, we
have 10 · 12 = 120 pieces shaded. Thus, these fractions are equivalent.
In this example, we chose a factor to multiply the numerator and denominator of each
side by. We chose the factor for the left-hand side to be the right-hand side’s denominator,
and vice-versa.
This illustrates a pattern that appears in the Fraction Simplification Theorem, Theorem
2.2.1.
Example 2.2.8. Suppose that
a c
= ,
b d
a c
where b and d are two given fractions. Following the same idea we had above, we will find
common denominators so that we can compare numerators.
a ad c bc
= and = .
b bd d bd
Thus, because the two fractions are equal and now have the same denominator, we must
have ad = bc.
This idea is often taken as the definition of fraction equivalence; here, we will present it
as a theorem.
a c
Theorem 3 (Fraction Equivalence). Two fractions b
and d
are equivalent if and only if
ad = bc.
Example 2.2.9. The fractions 64 and 23 are equivalent since 4 · 3 = 12 = 6 · 2. In terms of the
Fraction Simplification Theorem, this looks like
4 4·3 2 2·6
= and = .
6 6·3 3 3·6
22
Then we are just comparing the numerators 4·3 and 6·2, which is precisely what the theorem
tells us to do!
3 4 3
Example 2.2.10. The fractions 6
and 9
are not equivalent since 3 · 9 6= 4 · 6. Also, 6
is 0.5
and 49 = 0.4 is less than 0.5.
16 25
Example 2.2.11. The fractions 30
and 45
are not equivalent since 16 · 45 = 720 and 30 · 25 =
750.
4
Example 2.2.12. The fractions −5
and −4
5
are equivalent since 4(−5) = (−4)(5).
x x·6 6x
= = .
3 3·6 18
Thus for 6x
18
= 12
18
, we’d need that 6x = 12 so that x = 2.
x 12
Solution 2. Using the Fraction Equivalence Theorem, we’d have 3
= 18
if and only if
x · 18 = 3 · 12. Thus, we’d need 18x = 36 so that x = 2.
6 2 45 15 3
2. 15
and 5
6. 60
and 20
10. −3
7
and −7
9 3 28 14 21
3. 12
and 4
7. 40
and 20
11. 15
and −7
−5
5 25 33 3 4(x+y) 4
4. 6
and 30
8. 55
and 5
12. 11(x+y)
and 11
23
Show that the given fractions are equivalent using the Fraction Equivalence Theorem.
12 8 30 12 x+1 1
13. 15
and 10
17. 35
and 14
21. 3(x+1)
and 3
4 20 121 33
14. 6
and 30
18. 44
and 12 22. 1
and b
8 8b
12 9 4
15. 16
and 12
19. −4
15
and 23. 5x
and 5
−15 x
24 37
16. −110
55
and −2 20. −24
−31
and 31
24. 1
and 37
Each diagram illustrates a certain fraction. Subdivide each diagram to show that the fraction
it represents is equivalent to the given fraction.
9
25.
12
12
26. 18
15 29.
48
8
27.
20
10
28.
14 32
30.
72
a 1
31. Explain why ab
= b
for any integers a and b, provided b 6= 0.
ab
32. Explain why b
= a for any integers a and b, provided b 6= 0.
Notice that before we can simplify, we must factor both the numerator and denominator.
36
Example 2.3.3. Simplify 48
.
Solution:
36 3 · 12 3
= = .
48 4 · 12 4
The previous example illustrates a useful time-saving device: if the numerator and de-
nominator share a common factor at all (in any order), it may be removed from both.
25
x · (x − 2)
Example 2.3.7. Simplify the fraction .
(x − 2)(x + 3)
Solution. First note that for the fraction to make sense, the denominator must not
equal zero. Thus we have the implicit assumptions that x 6= 2 and x 6= −3.
Using the Fraction Simplification Theorem, we have
x · (x − 2) x · (x − 2) x
= = , x 6= 2.
(x − 2)(x + 3) (x + 3)(x − 2) x+3
Note that in all of the above fractions we have the implicit assumption that x 6= −3, but
for the final expression to equal the previous two expressions, we need the explicit assumption
that x 6= 2 since it is no longer clear that this restriction applies.
256x
Example 2.3.8. Simplify the fraction .
120x2
Solution. In order to apply the Fraction Simplification Theorem, we must have the
numerator and denominator factored:
256x 4 · 2 · 32 · x 32 32
2
= = =
120x 4 · 2 · 15 · x · x 15 · x 15x
Note that the assumption that x 6= 0 is implicit in all of the expressions above.
The moral of this section is twofold: first, fraction equivalence arises from the very natural
context of subdivision. Second, before you can simplify, you must factor!
33. At Rita’s high school, 600 out of 900 students are female. What (simplified) fraction
of the students are female? What percentage of the students are female?
34. In a survey, 800 out of 1000 dentists preferred NoCav brand toothpaste to the alter-
native. What (simplified) fraction of dentists preferred NoCav? What percentage of
dentists preferred NoCav?
Example 2.4.1. Figures for 83 and 82 are shown below. Remember what these fractions mean:
the pie has been divided into 8 equal pieces; We begin with 3 of those and then gain 2 more,
so we end up with 5; the figure supports this.
27
+ =
3 2 5
+ =
8 8 8
This indicates how we define addition of fractions that have a common denominator.
a b
Definition 2.4.1. Let c
and c
be fractions. Then
a b a+b
+ = .
c c c
Example 2.4.2. This time we will apply the definition above and look at a fraction strip
model, as well. The exercise is to add 59 and 92 . According to the definition, this should be
5 2 5+2 7
+ = = .
9 9 9 9
To see that this is reasonable, consider the fraction strip models for 95 (top strip) and 29
(bottom strip). We have lined them up so that we can just count the shaded portions from
left to right.
5 2 7
Figure 2.13: + =
9 9 9
Example 2.4.3. The same idea holds when the sum is greater than a whole unit.
7 9 7+9 16
+ = = .
12 12 12 12
7 9
In the figure, 12
is represented by the top strip, and 12
is represented by the bottom
strip.
1 whole unit
7 9 16
Figure 2.14: + =
12 12 12
28
Notice that the shaded portion extends four squares beyond the full unit, so
16 4
=1+ .
12 12
NOTE: It is important to recognize that the fraction bar is also another grouping symbol.
When you see
a
,
b
you should think of it as
(a)
.
(b)
That is, the numerator is a group, the denominator is a group, and the fraction itself is a
group. Thus,
3+9 (3 + 9)
is ,
16 16
and not
3 9
+ 9 or 3 + .
16 16
The addition must be performed before the division because of the grouping.
Example 2.4.4. So far we have only considered positive fractions, but the definition of addi-
tion applies whether the fractions are positive or negative.
Example 2.4.5.
−5 9 (−5) + 9 4
+ = = .
11 11 11 11
Since subtraction is really a form of addition, we have the following theorem about
subtraction of fractions that have the same denominator.
a b a b a−b
Theorem 4. Let c
and c
be fractions. Then − = .
c c c
Example 2.4.6. Consider 65 − 62 . First, we notice that we have divided the unit into 6 equal
pieces. The subtraction indicates that we begin with 5 of these and take away 2, so we
expect to be left with 3. The figure below illustrates this, and the computation using the
theorem agrees as well. In the figure, the diagonal slashes represent the 2 shaded regions we
are removing.
29
5 2 3
Figure 2.15: − =
6 6 6
5 2 5−2 3 1
− = = = .
6 6 6 6 2
Example 2.4.7.
5 −3 5 − (−3) 8
− = = = 1.
8 8 8 8
3 2
Figure 2.16: and
7 5
For a final answer, we need to know the total length of the shaded portions, but since the
shaded pieces are of different sizes, it is very difficult to tell what that length is! We really
need to arrange things so that the pieces are all the same size.
If we divide each piece of the top unit into 5 equal pieces and each piece of the bottom
unit into 7 equal pieces, both units will be in 35 equal pieces. Now we can just count the
total number shaded, as before.
3 15 2 14
Figure 2.17: = and =
7 35 5 35
This subdividing is the same thing we did before to discuss equivalent fractions. Here, we
are finding equivalent fractions that have the same denominators because we already know
30
3 2 15 14
+ = +
7 5 35 35
15 + 14
=
35
29
= .
35
Example 2.4.10. In the previous example, we could have found a smaller common denomi-
nator. Since
4 4·5 20 8 8·3 24
= = and = = ,
6 6·5 30 10 10 · 3 30
we have
4 8 20 24
+ = +
6 10 30 30
20 + 24
=
30
44
= ,
30
22
which will also reduce to 15
.
31
These principles apply even if you don’t know what the actual numbers in the fraction
are.
Example 2.4.12.
a b a·5 b·2
+ = +
2 5 2·5 5·2
5a + 2b
= .
10
Example 2.4.13.
a c ad bc
+ = +
b d bd bd
ad + bc
= .
bd
The prior examples illustrate the following theorem. You should study the Fraction
Addition Theorem until the pattern is clear to you.
a c
Theorem 5 (Fraction Addition). Let b
and d
be fractions. Then
a c ad + bc
+ = .
b d bd
32
4 5 4(8) + 7(5)
+ =
7 8 7(8)
32 + 35
=
56
67
= .
56
The theorem automatically takes care of finding a common denominator.
Example 2.4.15.
7 12 7(5) + (−6)(12)
+ =
−6 5 (−6)(5)
35 − 72
=
−30
−37
=
−30
37(−1)
=
30(−1)
37
= .
30
3
Example 2.4.16. In this example, we need to convert 3 to the fraction 1
in order to use the
Fraction Addition Theorem.
6 3 6
3+ = +
13 1 13
3 · 13 + 6 · 1
=
1 · 13
45
= .
13
33
Example 2.4.17.
x 6x x · (7) + 6x · y
+ =
y 7 y · (7)
7x + 6xy
= .
7y
Example 2.4.18.
x 6x x · z + 6x · z
+ =
z z z·z
xz + 6xz
=
z2
z · (x + 6x)
=
z·z
7x
= .
z
This example shows that the Fraction Addition Theorem works even if we already have
a common denominator.
1. 4.
2. 5.
3. 6.
3 2 3 8 31 17
7. 8
+ 8
13. 10
− 10
19. 40
− 20
4 2 13 6 21
8. 15
+ 15
14. 22
− 22
20. −15
8
+ 12
8 2 23 11 2
9. 10
+ 10
15. 80
− 80
21. 7 + 5
6 3 38 12 1
10. 12
+ 12
16. 56
− 56
22. 3 − 9
10 5 3 2 15 30
11. 21
+ 21
17. 5
+ 15
23. 8
+ 7
14 5 5 8 5 8
12. 20
− 20
18. 7
− 12
24. 6
+ 11
37. A board, known as a 2×4, measures 72 inches wide by 23 inches deep. If two such boards
are nailed together in an L-shape (so that the depth of one is nailed to the width of
the other), what is the total width of the L?
38. James’ stock rose 5 18 on Monday and fell 2 38 on Tuesday. What was the net change in
value?
39. Sarah installed baseboards in her house that were 3 58 inches tall. On top of the base-
9
boards, she put molding that extended an additional 16 of an inch. What is the
combined height of the baseboards and molding?
6 2 6
of .
8 3 8
2 6 4
Figure 2.18: of is .
3 8 8
2 6
By counting the heavily shaded portions, we can see that 3
of 8
is 48 .
The “pie model” for fractions is not very convenient for finding fractions of fractions.
Instead, we will return to a rectangular model.
Example 2.5.2. What is 54 of 73 ?
Solution: To determine this, we first need a diagram that illustrates 37 .
3
Figure 2.19:
7
36
The shaded portion is 37 , so we need to find 45 of that. To accomplish this, we will divide
the unit (the large rectangle) into 5 equal pieces. This will also subdivide each of the small
shaded rectangles into 5 equal pieces.
3
Figure 2.20: and the unit divided into 5 equal pieces
7
Finally, we will shade 4 of the 5 horizontal rectangles. This causes some of the small
squares to be shaded twice; they are shaded darker in the figure. The darkly shaded portion
represents 45 of 73 . Notice also the symmetry in the figure; 37 of 45 would look just the same,
so these are equal quantities.
4 3
Figure 2.21: of
5 7
If you just consider the portion that was originally shaded, 37 , you can see that that was
subdivided into 5 equal pieces, and 4 of those were shaded more darkly. We can count that
there are 12 darkly shaded squares, or we can multiply 3 · 4 = 12 since each of the 3 vertical
rectangles contributed 4 squares to the darkly shaded area. Similarly, the entire rectangle
was divided into 35 equal pieces since each of the 7 original vertical rectangles contributed
5 squares to the total: 7 · 5 = 35.
Overall, then, we have that 54 of 73 is 12
35
. In hindsight, we could have found this by
multiplying numerators and denominators:
4 3 4·3 12
of is = .
5 7 5·7 35
3
Figure 2.22:
4
To find 52 of the shaded region, we must subdivide it into 5 equal pieces and shade 2 of
those. In fact, we again subdivide the original unit into 5 equal pieces and shade 2 of those;
this accomplishes the same thing, but also allows us to see how many of the resulting small
squares it takes to fill the unit. The darkly shaded portion is the final result.
2 3
Figure 2.23: of
5 4
Since each of the 4 vertical rectangles contributes 5 squares to the full unit, the unit is
made up of 4 · 5 = 20 squares. Since each of the 3 shaded vertical rectangles contributes 2
to the darkly shaded region, there are 3 · 2 = 6 squares in there. Thus, we have that 25 of 34
6 3
is 20 = 10 , as before.
Example 2.5.4.
7 5 7·5
· =
12 3 12 · 3
35
= .
36
38
The definition of multiplication applies to any fractions; the numerator and denominator
need not be positive.
Example 2.5.5.
−3 5 (−3)(5)
· =
2 −9 2(−9)
−15
=
−18
5(−3)
=
6(−3)
5
= .
6
a b
Definition 2.5.2. If b
is a nonzero fraction, then a
is the reciprocal of ab . Likewise, a
b
is
the reciprocal of ab .
2
Example 2.5.6. The reciprocal of 5
is 52 . Notice that
2 5 2·5
· =
5 2 5·2
10
=
10
= 1.
The result in the previous example is true in general: the product of reciprocal fractions
is 1.
Example 2.5.7.
7 11 7 · 11
· =
11 7 11 · 7
77
=
77
= 1.
39
Example 2.5.8.
a b ab
· =
b a ba
ab
=
ab
= 1.
The question now is, how do we divide by fractions? The idea of division is to split up a
number into equal pieces that add up to that number. For example, when we divide 12 by
3, we are asking, how many groups of size 3 are there in 12? Similarly, when we divide 43 by
1
8
, we are really asking, how many groups of size 18 are there in 43 ?
Example 2.5.10. How many groups of size 18 are there in 43 ?
Solution: The strip models below show 34 and 81 .
3 1
Figure 2.24: and
4 8
1
With these particular fractions, it is not hard to see that it will take 6 pieces of size 8
to
make 43 . Thus,
3 1
÷ = 6.
4 8
4
Example 2.5.11. What is 5
÷ 23 ?
4 2
Solution: We have fraction strips for and below.
5 3
40
4 2
Figure 2.25: and
5 3
It is much more difficult to see how many pieces of size 32 there are in 45 because the sizes
of the pieces are not the same. However, if we divide each piece of the 45 model into 3 equal
pieces and each piece of the 32 model into 5 equal pieces, we will be able to compare them
more readily. (This is similar to what we did with our strip model for addition.)
4 12 2 10
Figure 2.26: = and =
5 15 3 15
Now we can see that the 32 has been broken into 10 equal pieces: each of the 2 original
shaded pieces contributed 5 pieces when subdivided, and 2 · 5 = 10.
It takes 12 of those small pieces to make up 45 : each of the original 4 shaded pieces
contributed 3 pieces when subdivided, and 4 · 3 = 12.
That is, if we break 32 into 10 equal pieces, it takes exactly 12 of those to make 54 .
Therefore,
4 2 4·3 12
÷ = = .
5 3 2·5 10
It is important to observe here that 12
10
12
is not 10 of the original unit, but of 32 . (That is,
after all, what the question was!) We can check that we have done this correctly: 12 10
of 32 is
12 2 12 · 2
· =
10 3 10 · 3
24
=
30
4·6
=
5·6
4
= ,
5
which is what we expect.
41
Example 2.5.13. Recall that the fraction bar is another way of expressing division. Thus,
3
11 3 2
2
= ÷
5
11 5
3 5
= ·
11 2
3·5
=
11 · 2
15
= .
22
Example 2.5.14.
1 3
3 = 1÷
8
8
8
= 1·
3
8
= .
3
1
Thus, 3 is just the reciprocal of 83 .
8
42
Example 2.5.15.
1 a
a =1÷
b
b
b
=1·
a
b
= .
a
1
Again, we just get that a the reciprocal of ab .
b
7
Example 2.5.16. We can also think about division algebraically. What is 8
÷ 25 ? Let us
temporarily call the answer ab since we don’t know yet. Thus,
7 2 a
÷ = ,
8 5 b
7 5 a 2 5
· = · ·
8 2 b 5 2
a
= ·1
b
a
= .
b
2 2
x x
6 =
6
5x 5x
2 6
= ÷
x 5x
2 5x
= ·
x 6
2·5·x
=
x·2·3
5
= .
3
2
1. 3
of 52 . 6. 9
10
of 150.
4
2. 5
of 81 . 7. 4 divided by 12 .
2 5
3. 3
of 12
. 8. 4 divided in half.
1
4. 2
of 21 . 9. 6 divided by 31 .
4
5. 3
of 87 . 10. 10 divided by 43 .
4 41 14
12. 3
14. 133
16. −71
38
18. 15
Compute as indicated, simplifying where possible. Be sure to include any implicit or explicit
assumptions you may need.
3 2 11 9 4
19. 4
· 9
23. 7
· 12
27. 5 · 5
1 6 45 4 12
20. 3
· 5
24. 4
· 45
28. −7 · 7
7 8 4 7
21. 8
· 3
25. 5
· 10 29. 15 · 3
3 4 3
22. 13
· 14
26. 32
· 16 30. −4
21
· (−14)
Compute as indicated, simplifying where possible. Be sure to include any implicit or explicit
assumptions you may need.
−10 7 8z 45
31. · 34. ·
21 −5 15 4(z − 6)
x+1 3 5 2t
32. · 35. ·
4 −(x + 1) 2t + 3 5
3 x −5(x + 2) 4(x + 1)
33. · 36. ·
x 3 x+1 2x + 1
Compute as indicated, simplifying where possible. Be sure to include any implicit or explicit
assumptions you may need.
1 2 12 9 5 15 2
37. 4
÷ 5
41. x
÷ x
45. 5 ÷ 2
49. 18
÷ 3
2 3 4 4 3
38. 3
÷ 4
42. 7
÷ 7
46. 6 ÷ 8 50. −15
71
÷ −15
32
1 3 3 8 4 31 31
39. 5
÷ 5
43. 8
÷ 3
47. −3
−12
÷ 9 51. 17
÷ 19
7 15 2 44 3
40. 11
÷ 11
44. −4
5
÷ 9
48. 16
÷ 32
52. −23
4
÷8
45
Compute as indicated, simplifying where possible. Be sure to include any implicit or explicit
assumptions you may need.
4x + 15 −3 x(x + 3) 3(x + 3)
53. ÷ 55. ÷
12 − 2x x(6 − x) 4(2x − 1) x
x x (x + 1)(x + 2) (x + 1)
54. ÷ 56. ÷
x+1 5 (x + 4)(x + 5) (x + 4)
Compute as indicated, simplifying where possible. Be sure to include any implicit or explicit
assumptions you may need.
5 3
57. x
+ x+1
63. −4
3
+ −10
7
8
58. 11
− −3
11 64. 3
·4
z
21 17
59. 17
· 5 10 2x
65. 9
· x+2
17
60. 15 − 3
65 7
61. 2
÷ 4 66. 16
+ 3
3x−18 x−6
12
62. −32
4
÷8 67. −64
15
− −38
46
Compute as indicated, simplifying where possible. Be sure to include any implicit or explicit
assumptions you may need.
3 5 14 9 x+1
7 11 x+2 11 2x+1
68. 2 70. 3 72. 3
74. 9 76. 5
5 11 x+2 38 3
3 2 5 51 4
8 x 7 118 7
69. 5 71. 9 73. 5 75. 51 77. −4
8 x 12 96 7
79. Marcus works at the bookstore two days out of every five all year through. If there are
365 days in a year, how many days in a year does Marcus work?
80. Marcy estimated that each person attending her party would eat 3 slices of pizza,
where each pizza is cut into 8 slices. If Marcy has 24 guests, how many pizzas will she
need?
1
81. Marcy can afford to buy 15 pizzas. If she expects each guest to eat 3
of a pizza, how
many guests can she invite?
Example 2.6.2. A cookie recipe calls for 1 34 cups of flour. How many cups are necessary to
make 2 21 batches?
Solution: First, notice that 1 34 = 47 and 2 12 = 25 . Each batch requires 74 cups, and we
need 25 of those; thus, we need
5 7 35
· =
2 4 8
3
cups, or 4 8 cups. (35 divided by 8 is 4 with a remainder of 3.)
47
Example 2.6.3. Sarah earns $37,500 per year. If she receives a 5% raise, how much will she
earn per year?
Solution: A 5% raise means that she will make her salary plus an additional 5% of her
5
salary, so we need to compute 5% of $37,500. Remember that 5% = 100 , so we need to know
5
100
of $37,500. This is
Definition 2.6.1. Two changing quantities are said to be proportional if their ratio is
constant. If a and b are proportional, then a = kb for some constant k, called the constant
of proportionality.
a
Notice that if a is proportional to b, then the ratio b
= k, for some constant k.
Example 2.6.4. If a person is travelling a constant speed, then the distance travelled is
proportional to the time elapsed. That is, if d is the distance travelled and t is the time
elapsed, then
d = rt
for some constant r (the speed, or rate). For example, if Jim travels at a constant rate of 55
miles per hour, then
d = 55t,
where d is Jim’s distance in miles and t is the time elapsed in hours.
Thus, after 1 hour, Jim has travelled d = 55(1) = 55 miles; after 2 hours, Jim has travelled
d = 55(2) = 110 miles; and after 3.75 hours, Jim has travelled d = 55(3.75) = 206.25 miles.
Example 2.6.5. A certain car can travel 450 miles on 18 gallons of gasoline. At this rate,
how many gallons of gasoline will be consumed on a 1,000 mile trip?
Solution 1. Let x denote the number of gallons of gasoline consumed on the 1,000 mile
trip. Then the ratio of miles per gallon is constant so that
450 1000
= .
18 x
Using the Fraction Equivalence Theorem, we can solve this proportion as follows:
48
1000 x
= Given
450 18
1000 · 18 = 450 · x Fraction Equivalence
450 18
= Given
1000 x
450 · x = 1000 · 18 Fraction Equivalence
Notice that we’ve presented four correct solutions to the above example. In general, a
given proportion will yield several equivalent equalities:
a c
= Given Proportion
b d
ad = bc Fraction Equivalence
a b
= Fraction Equivalence
c d
c d
= Fraction Equivalence
a b
b d
= , Fraction Equivalence
a c
provided each of a, b, c, and d are nonzero so that each fraction makes sense.
Example 2.6.6. A pine tree casts an 11-foot shadow. At the same time, a woman, who is 5
feet 4 inches tall, casts a 16 inch shadow. Determine the height of the pine tree.
Solution 1. Let x denote the height of the pine tree (in feet). Converting the other
measurements to feet we have that the woman is 5.3 feet tall and casts a 1.3 foot shadow.
x ft
5.3 ft
C A
1.3 ft
11 ft
At a given time, the ratio of height to shadow length is constant; we also say that the
triangles shown are similar. Therefore, we have the proportion
x 11
= Given Proportion
5.3 1.3
x · 1.3 = 5.3 · 11 Fraction Equivalence
Solution 2. Let x denote the height of the pine tree (in feet). Converting the other
measurements to (fractional) feet we have that the woman is 5 13 = 16
3
feet tall and casts a
1 4
1 3 = 3 foot shadow. Thus, we have the proportion
x 11
16 = 4 Given Proportion
3 3
x · 34 = 16
3
· 11 Fraction Equivalence
4x 176
= Multiplication
3 3
4x = 176 Multiply by 3
x = 44 Divide by 4
x = 44 Multiplication
13. A car consumes 7 gallons of gasoline on a 250-mile trip at 55 mph. How much gasoline
does it consume on a 450-mile trip at the same speed?
51
14. The rate that water from a hose fills a swimming pool is proportional to the amount
of time elapsed. If it takes 12 minutes for 150 gallons, how long will it take for 6000
gallons?
15. The radius of a circle is proportional to its circumference. If a circle with a circumfer-
ence of 12 meters has a radius of approximately 1.90986 meters, what (approximately)
is the radius of a circle with a circumference of 20 meters?
16. A building casts a 120-foot shadow at 9 AM. Also at 9 AM, a 50-foot flagpole casts a
30-foot shadow. How tall is the building?
17. The weight of an object (at the surface of the earth) is proportional to its mass. If
an object with a mass of 35 kg weighs 343 Newtons, how much does a mass of 75 kg
weigh? (Newtons are a metric measurement of force, or, in this case, weight.)
18. The force exerted by a spring is proportional to how far it is stretched or compressed.
If the spring is stretched 8 inches, it exerts a force of 45 pounds. What force does it
exert if it is only stretched 3 inches?
20. If 3 pizzas can feed 8 children, how many children can 12 pizzas feed?
21. The thickness of a stack of paper is proportional to the number of sheets in the stack.
If a stack of 500 sheets is 2 inches thick, how thick is a stack of 1250 sheets?
22. The amount of paint needed to paint a room is proportional to the area of the walls.
If a room with 1000 square feet of walls space requires 2.5 gallons of paint, how many
gallons of paint are needed for a room with 1750 square feet of wall space?
23. Scale models are models of objects whose measurements are in proportion to the actual
objects. A scale model of a jet is 15 inches long. If the constant of proportionality is
80, how long is the actual jet?
24. A scale model of a race car has a wheel that is 1.5 inches in diameter; the wheel of the
actual race car has a diameter of 24 inches. If the model is 12 inches long, how long is
the actual race car?
Similar triangles are triangles whose corresponding sides are in proportion, as in Example
2.6.6. A theorem from Geometry states that triangles are similar if two angles of one triangle
are the same as two angles of the other triangle. (In Example 2.6.6, both triangles had a
right angle, and they shared the angle at A.) For the following problems, (a) verify that the
triangles in each pair are similar and (b) use a proportion to find the missing length.
29
5
x x
8
25. 12 26. 24 12
52
31.875
17 x
27. 8
Definition 2.7.1. A rational expression is a fraction that may (or may not) include a
variable (or variables) in the numerator and/or denominator.
Since rational expressions are defined in terms of fractions, we will apply the same def-
initions for operations, equivalence, and simplification as we did for fractions. Notice that
this is much more abstract now: it doesn’t mean much to say that we are dividing a unit up
into x + 5 equal pieces, so the rational expression
x−3
x+5
must be taken as just an object in its own right. The important thing to remember is that
it is analogous to a regular fraction, and it will behave the same way.
Since rational expressions are modelled after fractions, they are simplified in the same
way as fractions.
Example 2.7.2.
(3x + 2)(5x − 7) 3x + 2 7
= , x 6=
(3x − 2)(5x − 7) 3x − 2 5
The numerator and denominator share the factor of 5x − 7. When this common factor is
“cancelled,” the assumption that x 6= 75 must be explicitly made. Note that the assumption
that x 6= 23 is implicit in both expressions; we can see that we must make this assumption
by considering the denominator.
53
Example 2.7.3.
x(x + 2) x+2
= , x 6= 0
x(x − 3) x−3
Example 2.7.4.
x+2
x+4
cannot be simplified since the numerator and denominator do not have a factor in common.
That x 6= −4 is implicit for the expression to be defined.
Example 2.7.5.
x−5 x−5
=
4x − 20 4(x − 5)
1(x − 5)
=
4(x − 5)
1
= , x 6= 5.
4
Notice that we had to apply the distributive law from Chapter 1 to the numerator before we
could simplify. The distributive law will be covered in detail in Chapter 4.
Example 2.7.6.
x(x + 2) − 4(x + 2) (x − 4)(x + 2) x+2
= = , x 6= 4
(x − 4)(3x + 5) (x − 4)(3x + 5) 3x + 5
Example 2.7.7.
4(5 − x) 4(5 − x) 5−x
= = .
12(5 + x) 4 · 3(5 + x) 3(5 + x)
All of the operations on rational expressions are defined by analogy with operations on
fractions since they are set up in terms of fractions.
p q
Definition 2.7.2. Let r
and r
be rational expressions with a common denominator. Then
p q p+q
+ =
r r r
and
p q p−q
− = .
r r r
54
p r
Definition 2.7.3. Let q
and s
be rational expressions. Then
p r ps + qr
+ =
q s qs
and
p r ps − qr
− =
q s qs
3 4x
Example 2.7.8. Since x
and x
have the same denominator, we may use the first definition
to get
3 4x 3 + 4x
+ = .
x x x
x+2 5−x
Example 2.7.9. Since x
and x
have the same denominator, we may use the first definition
to get
x+2 5−x (x + 2) + (5 − x) 7
+ = =
x x x x
since the x and the −x cancel each other. Note that the only necessary assumption is the
implicit assumption that x 6= 0.
Notice that (x + 2) and (5 − x) were each grouped in the middle expression. Remember,
the fraction bar acts as a grouping symbol, and we need to keep that in mind when we
compute with rational expressions.
Example 2.7.10.
4
Example 2.7.11. Since −1
x
and x+2
have different denominators, we will use the second defi-
nition.
3x − 2
= ,
x(x + 2)
and we arrive at the same result as before.
Example 2.7.12.
1 5 1(x − 2) − 5(x + 3)
− =
x+3 x−2 (x + 3)(x − 2)
x − 2 − 5x − 15
=
(x + 3)(x − 2)
−4x − 17
= .
(x + 3)(x − 2)
Example 2.7.13.
4x − 7 3x + 1 4x − 7
3x + 1 + = +
6 1 6
(3x + 1)(6) + 1(4x − 7)
=
(1)(6)
18x + 6 + 4x − 7
=
6
22x − 1
= .
6
p r
Definition 2.7.4. If and are rational expressions, then
q s
p r pr
· = .
q s qs
56
p r r
Definition 2.7.5. If and are rational expressions and 6= 0, then
q s s
p r p s
÷ = · .
q s q r
These operations are again defined in terms of the same operations on fractions.
Example 2.7.14.
x x+2 x(x + 2) x
· = = , x 6= −2
x+2 x−1 (x + 2)(x − 1) x−1
Since x 6= 1 is implicit in all of the above expressions, it need not be specified. That
x 6= −2, on the other hand, must be made explicit for the equality to hold.
Example 2.7.15.
(x + 3)(x + 2) (3x + 4)(x − 1) [(x + 3)(x + 2)][(3x + 4)(x − 1)]
· =
(x − 2)(3x + 4) (x + 3)(x + 2) [(x − 2)(3x + 4)][(x + 3)(x + 2)]
x+1 4
= , x 6= − , x 6= −3, x 6= −2.
x−2 3
Notice, again, that the implicit assumption x 6= 2 is implied in all of the above expressions
and need not be specified.
Example 2.7.16.
x(x − 3) x(x + 1) x(x − 3) (2x − 3)(x + 2)
÷ = ·
x+2 (2x − 3)(x + 2) x+2 x(x + 1)
(x − 3)(2x − 3)
= , x 6= −2, x 6= 0.
x+1
Example 2.7.17.
(x + 1)(x − 3) (3x + 2)(2x + 6) (x + 1)(x − 3)(3x + 2)(2x + 6)
· =
3x + 2 x+1 (3x + 2)(x + 1)
= (x − 3)(2x + 6).
57
Example 2.7.18.
(3x+4)(x−5)
(2x+1)(x+2) (3x + 4)(x − 5) (3x + 4)x
(3x+4)x
= ÷
(2x+1)(x−2)
(2x + 1)(x + 2) (2x + 1)(x − 2)
(x − 5)(x − 2)
= .
(x + 2)x
Example 2.7.19.
(x + 5)(x − 2)
=
(1)(3x + 1)
(x + 5)(x − 2)
= .
3x + 1
Example 2.7.20.
4x + 9 3x − 17 4x + 9
(3x − 17) · = ·
3x − 17 1 3x − 17
(3x − 17)(4x + 9)
=
(1)(3x − 17)
4x + 9
=
1
= 4x + 9
58
Example 2.7.21.
3x − 8 4x + 2 3x − 8
(4x + 2) ÷ = ÷
2x + 1 1 2x + 1
4x + 1 2x + 1
= ·
2 3x − 8
(4x + 1)(2x + 1)
= .
2(3x − 8)
Compute as indicated and simplify. Be sure to state any assumptions you may need.
3 7 5x + 2 5x + 2
13. + 19. ÷
4x 4x 3x x−3
5−x x+2 4x + 2 2x + 1
14. − 20. ÷
x+1 x+1 (x − 3)(x − 5) x−3
8t + 2 3 8x − 12 x − 32
15. + 21. ·
5 5 2x − 43 (2x − 3)(x + 1)
9z − 5 3z − 23 4 5
16. − 22. +
(z + 2)(z + 3) (z + 2)(z + 3) 3x 2x − 7
x x−2 (5x + 24)(3x − 8) (21x − 4)x
17. · 23. ·
x−2 x+1 x(x − 3) (3x − 8)(x + 2)
x(x + 1) x−1 x 5x + 2
18. · 24. −
3x(2x − 2) (x + 1)(4x − 1) 4 12
59
4x 2x − 14 15t(u + 1) 12t
25. + 33. ÷
6x 3x 20u(t + 1) u
50x 2x 28x + 14 (x − 5)(3x + 13)
26. ÷ 34. ·
35(11x − 3) 4x + 1 (x − 5)(3x + 13) 2x + 1
6 2 −12 7
27. − 35. +
x 12x + 15 2x − 9 11x + 3
t(8t + 1) (8t + 1)(15t − 2) 14 x−7
28. ÷ 36. ÷
t+8 6 8x + 6 4x + 3
t + 5 24t − 5 4x − 9
29. − 37. 3x − 6 +
8 12 12
yz(x + 3) (4z + 7)x 3x + 2
30. · 38. ÷ (8x − 2)
(y − 1)(4z + 7) y(x + 3)(z − 2) x−8
8x − 11 12 3x + 5
31. + 39. (4x + 5) ÷
(x + 3)(5x − 1) (x + 3) 7x − 1
3x + 1 2x − 2 2x + 4
32. − 40. (6x − 11) ·
4x 8x x−5
Compute as indicated and simplify. Be sure to state any assumptions you may need.
4x 8x(8x−5)
x+5 12x+2
41. 12x 44. 4x
x−5 6x+2
3x (4x−3)(3x−1)
(x+2)(3x−6) 3x−1
42. 9 45. 12x−9
x−2 (x+4)(3x−2)
51x−17 (t+15)(6t−5)
x(2x+1) (t−2)(t+4)
43. 12x−4 46. t+15
x(x−14) (t−2)t
60
Chapter 3
Exponents
61
62
for any real number a. This goes both ways, too: if you see 11, you can think of it as 111 if
that is helpful. (We will see as we progress that sometimes it is helpful!)
an = 52
=5·5
= 25.
When the exponent is two, it is often pronounced as “squared” instead of “second power.”
Thus, 52 can be pronounced either “5 to the second power” or “5 squared.” Note that a 5
by 5 square has area of 25 units; hence, the terminology 25 “square” units.
an = 25
= 2 · 2 · 2 · 2 · 2,
(−10)2 = (−10)(−10)
= 100.
Example 3.1.7. Suppose that n = 4 and a = xy. Here, the x and the y represent unknown
real numbers. We have
an = (xy)4
= (xy)(xy)(xy)(xy)
= (x · x · x · x)(y · y · y · y)
= x4 y 4.
We had to use some of the properties of real numbers here; specifically, we needed to use
that multiplication is both associative and commutative to put all of the x’s together and
all of the y’s together.
The previous example illustrates an important point: in terms of the order of operations,
exponents receive a higher priority than multiplication or division.
Example 3.1.11. What is −24 ? We must again appeal to convention here. The expression
−24 is similar in nature to 2 · 63 in that it can be written as −1 · 24 . This means that we have
−1 · 24 = −1(2)(2)(2)(2) = −16.
an ≥ 0.
This is reasonable since an even exponent of n means that we have an even number of
factors, all of which are the same. Thus, if we have a negative base, we end up with a product
of an even number of negative numbers multiplied together, so the product is positive.
We continue this section with a brief discussion of polynomial expressions.
an xn + an−1 xn−1 + . . . + a2 x2 + a1 x + a0 ,
Example 3.1.12. 4x3 + 7x − 5 is a polynomial in x. The degrees and coefficients of each term
are in the table below.
The constant term is −5, the leading term is 4x3 , the leading coefficient is 4, and the
degree of the polynomial is 3. Notice that since there is no x2 term, we can think of the x2
term as having a coefficient of 0.
1
Example 3.1.13. 4x2 + x
is not a polynomial in x since it does not have the form specified
in the definition.
Example 3.1.14. −7t2 + 4t6 − 11t5 + t3 is a polynomial in t. The table below summarizes the
information for this polynomial. To make it easier to gather all of the information, we will
rewrite the polynomial in descending order of the exponents:
In polynomial expressions, terms of the same degree are referred to as like terms.
Example 3.1.15. In the polynomial expression
the terms 5x2 and 13x2 are like terms; −7x and 11 are not like terms.
Let’s briefly discuss one more use of exponents. In arithmetic, we learn to (prime-)factor
natural numbers; for example,
72 = 2 · 2 · 2 · 3 · 3.
This can be expressed much more compactly as
72 = 23 · 32 .
Such a factorization, with the prime factors listed in increasing order, is called the prime-
power representation of the number.
66
540 = 2 · 270
= 2 · 2 · 135
= 2 · 2 · 3 · 45
= 2 · 2 · 3 · 3 · 15
= 2·2·3·3·3·5
= 22 · 33 · 5.
Identify the expressions below that are polynomials and those that are not.
67
5 √
29. 31. x3 − 71x2 + 5x + 2 33. x+5
x
x 4 12 3
30. 32. x + 7x − 34. 2x + 4x − 1
5 5 x
Identify the degree, leading term, leading coefficient, and constant term of each polynomial.
5 4
35. 4x3 + 5x − 7x2 39. 15 43. 7
x − 83 x − 7
4
3x 8x 3x 15x x + 1 4x
47. · 49. · 51. +
5 x+1 10(x + 1) (x + 1) 3x 5
3x x 2x + 4 x − 5
48. − 50. ÷ x2 + 2x + 1 x
x + 1 2x − 3 x−5 x+2 52. −
2(x + 1) 2
2 3
(a) 4x5 (f) 3
x (k) 21x6 (p) − 27 x8
(b) −3x2 (g) − 58 x4 (l) −5.9x7 (q) 17x6
x8
(c) 15x5 (h) −15x8 (m) 36x4 (r) 3
6x5
(d) 5x2 (i) 31x3 (n) 5x7 (s) 7
3 5
(e) 4
x (j) −7x (o) 11x3 (t) −x
Example 3.2.2.
Example 3.2.3.
x4 · x1 = x4 · x
= (x · x · x · x) · x
= x·x·x·x·x
= x5 .
Example 3.2.4.
am · an = (a
|
· a ·{za · · · a})(a
|
· a ·{za · · · a})
m factors n factors
= a|
· a ·{za · · · a}
m + n factors
= am+n .
69
In each of the examples we just saw, we multiplied two powers of the same base (a)
together. The result was that we acquired factors of the base from each power, and the
exponent at the end was the sum of the original exponents. The theorem below summarizes
this.
Theorem 10 (Multiplication of Like Bases). Let a be a real number and let m and n be
natural numbers. Then
am an = am+n .
This theorem may be stated as follows: “To multiply like bases, add their exponents.”
Here, “like bases” means that the bases are the same.
Example 3.2.5. 53 · 54 = 53+4 = 57 . If we look at this in terms of the definition of exponents,
we see that
53 · 54 = (5 · 5 · 5)(5 · 5 · 5 · 5)
=5·5·5·5·5·5·5
= 57 ,
Example 3.2.6.
The same ideas apply to fractions and variables, as well as products with more than two
factors.
Example 3.2.7.
32 37 32 · 37
· =
53 52 53 · 52
32+7
=
53+2
39
= .
55
70
Example 3.2.8.
4 8
5 5 5 4+8
· =
6 6 6
12
5
=
6
5 12
We will see later how we can further simplify 6
.
Example 3.2.9.
x2 (x + 1)3 x6 (x + 1)11 = [x2 x6 ][(x + 1)3 (x + 1)11 ]
= x2+6 (x + 1)3+11
= x8 (x + 1)14 .
Example 3.2.10.
= (3.2)2+3+6+1
= (3.2)12 .
This computation illustrates the fact that we could have just added all of the exponents
in the beginning:
Because equality is symmetric, we may use the Multiplication of Like Bases theorem to
decompose a power, too.
7
Example 3.2.11. Simplify 339 .
Solution: In order to simplify a fraction, we need to find factors that the numerator and
denominator share. The numerator has a factor of 37 , and so does the denominator since
39 = 32+7 = 32 37 .
To see this, we have applied the Multiplication of Like Bases Theorem, Theorem 3.2.2,
“backwards.” Now we may compute.
71
37 37
=
39 32+7
37
= 2 7
3 3
1
=
32
1
= .
9
6
Example 3.2.12. Simplify 552 .
Solution: Notice that 56 = 52 54 . Thus,
56 52 54
=
52 52
4
=5
= 625.
Notice that the final exponent, 4, is the difference between the original exponents: 6 − 2 = 4.
Example 3.2.13.
x8 x8 1
12
= 4 8
= 4.
x xx x
Notice that the final exponent in the denominator, 4, is the difference between the original
exponents: 12 − 8 = 4.
In both of the preceding examples, the final exponent was the difference between the
original exponents. This makes sense: to simplify the fraction, we must cancel common
factors. If the exponent in the numerator is greater than that in the denominator, then the
entire denominator is cancelled, and what is left in the numerator is however many factors
the denominator did not cancel. This idea is recorded in the following theorem.
Theorem 11 (Division of Like Bases). Let a be a real number, and let m and n be natural
numbers. Then
8
>
> am−n if m > n
m <
a 1 if m = n
=>
an > 1
: if n > m
an−m
72
Example 3.2.14.
128
5
= 128−5 = 123 .
12
Example 3.2.15.
x14
= 1.
x14
Example 3.2.16.
(6x)3 1 1 1
7
= 7−3
= 4
= 4 4.
(6x) (6x) (6x) 6x
Example 3.2.17.
8x5 y 2 2x5−3 2x2
= = .
4x3 y 9 y 9−2 y7
Notice that in the last example, we had three different factors to worry about: a constant
factor, a factor involving x, and a factor involving y. However, we saw in Chapter 2 that we
can deal with each factor separately, and so that is what we did.
We now change gears slightly to consider another application of the Multiplication of
Like Bases Theorem.
Example 3.2.18. What is (45 )3 ?
Solution: To evaluate this expression, we need to expand the third power:
= [45+5 ](45 )
= 45+5+5
= 45·3
= 415 .
Thus, we have that (45 )3 = 45·3 = 415 . Notice why the exponent was 5 · 3: each of the
three factors of 45 had a 5 in the exponent, so the three fives were added together. Usually,
instead of adding three fives, we multiply 5 by 3.
73
Example 3.2.19.
= 57+7
= 57·2
= 514 .
Example 3.2.20.
Again, we end up with 5 fours in the exponent by using the Multiplication Of Like Bases
Theorem.
Example 3.2.21.
=am + m + m + . . . + m + m
= amn
Theorem 12 (Raising a Power (of a) to a Power). Let a be a real number and m and n
natural numbers. Then
(am )n = amn .
So far, most of our bases have had only a single factor. What happens when the base
has more than one factor?
Example 3.2.24.
(3 · 4)2 = (3 · 4)(3 · 4)
= (3 · 3)(4 · 4)
= 32 · 42 .
Example 3.2.25.
(3x)3 = (3x)(3x)(3x)
= (3 · 3 · 3)(x · x · x) (Multiplication is associative and commutative)
= 33 x3
Example 3.2.26.
(xy)4 = (xy)(xy)(xy)(xy)
= (x · x · x · x)(y · y · y · y)
= x4 y 4
Example 3.2.27.
Theorem 13 (Power of a Product). Let a and b be real numbers and n be a natural number.
Then
(ab)n = an bn .
Example 3.2.29.
The last example illustrates that sometimes you will need to use more than one property
to simplify an expression. Just keep the properties and the order of operations in mind as
you do!
Example 3.2.30.
= [1024x3·5 ][729x2·6 ]
= (1024x15 )(729x12 )
= 746496x15 x12
= 746496x15+12
= 746496x27
Example 3.2.31.
= 57 x12 x7
= 57 x12+7
= 57 x19
Since multiplication and division are very closely related, it seems reasonable that a
similar rule should hold for powers of quotients as does for powers of products.
76
Example 3.2.32.
3
4 4 4 4
=
7 7 7 7
4·4·4
=
7·7·7
43
= 3.
7
Example 3.2.33.
6
−2 −2 −2 −2 −2 −2 −2
=
9 9 9 9 9 9 9
(−2)(−2)(−2)(−2)(−2)(−2)
=
9·9·9·9·9·9
(−2)6
= .
96
Example 3.2.34.
4
2 2 2 2 2
=
3x 3x 3x 3x 3x
2·2·2·2
=
(3x)(3x)(3x)(3x)
24
=
(3x)4
16
=
34 x4
16
= .
81x4
Example 3.2.35.
5
x x x x x x
=
y y y y y y
x·x·x·x·x
=
y·y·y·y·y
x5
= .
y5
77
Example 3.2.36.
a n a a a
a a
= ···
b |
b b b {z b b}
n factors
n factors
z }| {
a · a · a···a · a
=
b| · b · b{z· · · b · b}
n factors
an
= .
bn
We record here for easy reference the rules we have developed thus far.
Theorem 15. Let a and b be real numbers and m and n natural numbers. Then
You should practice with these rules until they become comfortable. While you may
want to memorize them, it is far more important to understand them. That way, if you
forget the rule, you can figure it out again for yourself just by considering the definition of
exponents. Note that we will dramatically simplify the Division of Like Bases theorem in
the next section by introducing negative exponents.
As a final example, let’s consider how we can use these rules on rational expressions.
Example 3.2.39.
Notice that we used the Multiplication of Like Bases Theorem and the Division of Like
Bases Theorem.
79
8. [(−2)5 ]4 x2 + 1 5x + 6
27. −
3x3 15x2
9. (2x)4
14
7
t(t + 1)2
10. (5t) 28.
t+2
x 6
11. 29. 5x8 · x4 · x3
3
x4
3
12. (x y) 2 4 30. −x +
x+7
x4 t2 + 1 4
13. 31. ÷
x13 (t + 1) 2 (t + 1) (t2 + 1)3
5
5
(x3 + 3x + 1)4 32. (3x4 )3 (2x2 )5
14.
(x3 + 3x + 1)9
(3x)6 y 7 z 3 (x + 2)2
1 33.
15. 243 · 245 · 3x6 y 3 z 5 (x + 2)4
248
4 x3 (x2 + 4)3 x2 (x2 + 4)6
15 34. ÷
16. x(x2 − 2)4 x2 − 2
24
x 5x
3x2 (x+2)3 35. +
(x+1)2 (x2 +4) 2x + 1 x − 3
17. 9x5 (x+2)4 (x+1)
(x2 +4)4 (x + 2)3 (x + 2)4 (x + 1)
36. ·
(x + 1)(x + 3)2 (x + 3)2
x2 4x
18. − 4
x+2 x+8 x2 y 4
37.
19. [(x2 y 3)4 ]6 xy 6
Definition 3.3.1. Let a be a nonzero real number, and let n be a natural number.
1. a0 = 1.
1
2. a−n = .
an
1
3. = an .
a−n
At first glance, these definitions may not seem to help much, but we will see that they
are very convenient.
Example 3.3.1. Why should a0 = 1? Consider the following pattern:
24 = 16, 23 = 8, 22 = 4, 21 = 2, 20 = 1.
As the exponents go down, the value is cut in half each time so that when we get to an
exponent of 0, the value should be 1. In addition, notice that in the theorem if m = n, then
we have aam = 1, so if we use the Division of Like Bases idea and subtract the exponents, we
m
have am−m = 1, or a0 = 1.
1
Example 3.3.3. 4−3 = .
43
81
1
Example 3.3.4. = 35 .
3−5
Example 3.3.5.
−3 !3
2 1
= 2
5 5
3
5
=
2
53
=
23
125
= .
8
Notice that
2 −3 5 3
= .
5 2
That is, the negative exponent indicates a reciprocal.
Example 3.3.6.
x4 1
6
= 6−4
x x
1
=
x2
= x−2 .
Example 3.3.7.
t5 t5 1
= · −2
t−2 1 t
= t5 t2
= t7 .
Here, we separated the numerator and denominator by using the Fraction Multiplication
1
Theorem “backwards”. This allowed us to deal with the t−2 according to the definition.
Also, the final exponent 7 is equal to 5 − (−2), the difference in the original exponents.
82
Example 3.3.8.
5x2 y 6 1 5x2 y 6
=
x−3 y 6 x−3 y 6
= x3 · 5x2
= 5x3+2
= 5x5 .
We separated out the factor of x−3 in the denominator so that we could handle it separately.
Also notice that the final exponent 5 on x is 2 − (−3), once more the difference between the
exponents in the numerator and denominator.
Example 3.3.9.
x−3 x−3 1
= ·
x4 1 x4
1 1
= ·
x3 x4
1·1
=
x3 · x4
1
= .
x7
37
Example 3.3.10. 7 = 1. Notice that 30 = 1 as well, so the final exponent is once more the
3
difference between the exponents in the numerator and denominator: 7 − 7 = 0.
These examples indicate how the Division of Like Bases Theorem should be modified.
Theorem 16. Let a be a nonzero real number and let m and n be integers. Then
am
= am−n .
an
Example 3.3.11.
47
= 47−6
46
= 41
= 4.
83
Example 3.3.12.
27
= 27−9
29
= 2−2 .
Example 3.3.13.
x4 y 2 z x4 y 2 z 1
=
xy 5 z 3 x1 y 5 z 3
= x3 y −3 z −2 .
1511
= 1.
1511
We also have
1511
= 1511−11
1511
= 150
= 1,
so we see that this theorem is consistent with what we have done before.
The exponent laws we discovered in the last section hold for all integer exponents, not
just natural number exponents.
Theorem 17. Let a and b be real numbers, and let m and n be integers. Then
1. am an = am+n (Multiplication of like bases)
am
2. = am−n (Division of like bases)
an
3. (am )n = amn (Raising a power (of a) to a power)
Example 3.3.15.
Example 3.3.16.
= −1 · 45+(−6)
= −4−1
−1
= .
4
Example 3.3.17.
3 −4 3(−4)
−3 −3
=
11 11
−12
−3
=
11
12
11
=
−3
1112
=
(−3)12
1112
= .
312
85
Example 3.3.18.
−2 −2
3x3 3x3−1
=
5xy 5y
−2
3x2
=
5y
2
5y
=
3x2
(5y)2
=
(3x2 )2
52 y 2
=
32 (x2 )2
25y 2
=
9x2·2
25y 2
= .
9x4
Whew! That one took a lot of different properties.
x · x−1 = x1 · x−1
= x1−1
= x0
= 1,
1. 3−3 1 4
3. 5.
3−4 x−2
2. 5−2 4. x−3
86
for the largest such b that exists, if there is one. If there is such a b it is called the principal
nth root of a.
1
Example 3.4.1. 9 2 = 3 since 9 = 32 . Also, (−3)2 = 9, so which is the principal root? The
definition above says to take the largest number that works. Since only 3 and −3 satisfy
x2 = 9, the principal root is 3.
Notice that
1 1 1 1
9 2 · 9 2 = 3 · 3 = 9 and 9 2 + 2 = 91 = 9.
Also notice that 2
1 1
92 = 32 = 9 and 9 2 ·2 = 91 = 9.
1
Example 3.4.2. 121 2 = 11 since 121 = 112 . Again, (−11)2 = 121, but 11 is larger than −11,
so it is our principal root.
87
In both of the preceding examples, the exponent was 21 . We call a1/2 the (principal)
square root of a.
Example 3.4.3. 81/3 = 2 since 8 = 23 . Notice that 2 is the only number such that satisfies
x3 = 8, so there is no question about what the principal root is.
In the two preceding examples, the exponent was 13 . We call a1/3 the cube root of a.
1
Example 3.4.5. Suppose that x 3 = y. Then x = y 3 . Substituting, we see that
1
3
x = x3 .
In the previous sections, we saw that to raise a power of a base to another power, we
multiplied exponents. This still works here, too!
1
Example 3.4.6. (16x4 ) 4 = 2x if x is positive since
(2x)4 = 24 x4 = 16x4 ,
so
1 1
(16x4 ) 4 = [(2x)4 ] 4 .
We had to assume that x was positive in the above example in order to make sure that
we found the principal root. Throughout the rest of this section, we will assume
that variables are positive in order to simplify the exposition. If the variables are
allowed to be negative, the formulas are slightly more complicated. It is, therefore, extremely
important to keep in mind that all variables are assumed to be positive.
1
Example 3.4.7. (−4) 2 does not exist since there is no real number b such that b2 = −4.
1
(Recall that the square of any real number is at least 0.) Therefore, the symbol (−4) 2 is
undefined.
Example 3.4.9.
1 1
8 −3 =
2
since !3
−3
1 1
= 1 = 23 = 8.
2 2
It is not hard to check whether a given root is correct; for example, above, we just had to
−3
1
compute 2 and see whether we ended up with 8. But where does the number to check
come from? It comes from experience, educated guessing, and trial and error. You will need
to practice to become good at it, and we will also learn some rules to help us quickly identify
candidates.
Example 3.4.10.
1
(−8) 3 = −2 since (−2)3 = 8.
Example 3.4.11.
1
(x6 ) 3 = x2 since (x2 )3 = x2·3 = x6 .
Example 3.4.12.
1
x4 y 8 2
x2 y 4
=
16 4
since
2
x2 y 4 (x2 )2 (y 4 )2
=
4 42
x2·2 y 4·2
=
16
x4 y 8
= .
16
We will see that this definition allows us to keep all of the exponent properties we have
already discovered.
89
Example 3.4.13. 2
2 1
83 = 83 = 22 = 4.
Here, we used Example 3.4.3 for the second step.
Example 3.4.14.
2 1
2 1 1 1
x3x3 = x3 x3
2+1
1
= x3 Multiplication of Like Bases
3
1
= x3
3
= x3 Definition
= x1
= x.
Example 3.4.15. In the previous example, the exponents had a common denominator. When
1
we applied the definition of a rational exponent, that ended up giving us like bases of x 3 .
Here, however, the exponents do not have a common denominator, so we must first find one.
3 1 3 4
16 8 · 16 2 = 16 8 · 16 8
3 4
1 1
= 16 8 16 8
3+4
1
= 16 8 Multiplication of Like Bases
7
1
= 16 8
7
= 16 8 .
Example 3.4.16.
3 4 3·9 4·5
5 5 · 5 9 = 5 5·9 · 5 9·5
27 20
= 5 45 · 5 45
27 20
1 1
= 5 45 5 45
27+20
1
= 5 45 Multiplication of Like Bases
47
1
= 5 45
47
= 5 45 Definition.
90
Example 3.4.17.
4 2 4
2 1
27 3 = 27 3
2·4
1
= 27 3 Raising a Power to a Power
8
1
= 27 3
= (3)8
= 6561.
Example 3.4.18.
1
3 5 3
t4 = t 20
since
5 3 5
3 1
t 20 = t 20
h 1
i3·5
= t 20
h i15
1
= t 20
15
= t 20
3
= t4.
Notice that
3 1 3
· = .
4 5 20
Example 3.4.19.
3
3 1
4 4
(25x ) 2
= (25x ) 2
3
= [5x2 ] (See below)
= 53 (x2 )3 Power of a Product
= 125x6 .
In the second step, we get
1
(25x4 ) 2 = 5x2
91
because
(5x2 )2 = 52 (x2 )2
= 25x2·2
= 25x4 .
In the next example, we will compare two different methods for computing a quantity
raised to a rational power.
3
Example 3.4.20. What is 4 2 ?
Solution: 1 3
3
4 2 = 4 2 = 23 = 8.
On the other hand,
1 1
43 2 = 64 2 = 8,
which is the same result we found the first way.
1 5 − 1
1. 4 2 5. 64 6 8 3
9.
1 1
27
2. 8 3 6. 9 2
2
16
10. 125 3
1
3. 27 3 2 3
7. (−32) 5 16 4
2 11.
8 3 81
4. 1
8. (−8) 3
1
83 1
12. 121 2
93
Compute as indicated and simplify as much as possible. Assume that all variables are
positive.
1 2
13. (x4 ) 2 x− 5
20. 3
1
1 x
2
14. 16x 2 3
21. (x2 y) 4
3
15. (49x6 ) 2 15
1 1
2 7 22. x 6 y − 3
16. t t
3 3
5 1 − 1
17. y 8 y 8 x− 4 y 5
3 2 3
3 3 23. 5 6
18. x 5 x 4 x 8 y − 11
3
x4 1
19. 7 24. (25x4 y 6) 2
x 10
Definition 3.5.1. Let a be a real number, and let m and n be integers, with n 6= 0. Then
m √
a n = n am .
√
The symbol n is called a radical. With this notation, the integer n appearing in the
√
radical is called the index. The plural of index is indices. In the expression n a, a is called
the radicand.
2 √3
Example 3.5.1. x 3 = x2 .
√
3 1
Example 3.5.2. 8 = 8 3 = 2.
√
2 1
Example 3.5.3. 64 = 64 2 = 8. Usually, if the index is 2 we simply omit it. Thus,
√2
√
64 = 64,
Example 3.5.5.
Ì
1
4 x8 x8 4
=
y 12 y 12
1
x8· 4
= 1 Division of Like Bases
y 12· 4
x2
= 3.
y
Theorem 20. Let a and b be real numbers, and let m and n be natural numbers. Then
√ √ m √
1. n am = ( n a) if n a is defined.
√ √ √ √ √
2. n ab = n a n b if n a and n b are both defined.
È
a
√
na √ √
3. n
b
= √
n
b
if n
a and n
b are both defined and b 6= 0.
√
an = a if a is nonnegative.
4. n
√3
Example 3.5.6. 143 = 14.
È
Example 3.5.7. (41x)2 = 41x. (Recall: x ≥ 0.)
√ √
Example 3.5.8. 121 = 11 since 112 = 121. Also, − 121 = −11. The square root symbol
acts as a grouping symbol, too.
√
Example 3.5.9. 3
216 = 6 since 63 = 216.
95
√
3
Example 3.5.10. In order to find 3240, we first need to factor 3240. We have
3240 = 23 · 34 · 5.
Thus
√
3
√
3
3240 = 23 · 34 · 5
√
3
= 23 · 33 · 3 · 5
√3
√3
√3
= 23 33 3 · 5
√
= 2 · 3 15
√
= 6 15.
Notice how we used the properties of the theorem to separate the factors into pieces we could
handle.
Example 3.5.11.
√ √
125 = 52 · 5
√ √
= 52 5
√
= 5 5.
Here we first found the largest perfect square factor of 125 we could (namely, 25).
√
Example 3.5.12. x4 y 2 = x2 y. (Recall: x, y ≥ 0.)
Example 3.5.13.
√
3
√
3
x4 = x3 · x
√3 √
= x3 3 x
√
= x 3 x.
Notice that we needed to split the radicand into two pieces: the piece with a radicand of
x3 had an exponent that was divisible by the index, and the other had an exponent smaller
than the index. This is because of our definitions:
√ 1
x3 = (x3 ) 3
3
1
= x3· 3
= x1
= x.
Another way of looking at this is to observe that we found the largest perfect cube factor
of x4 , which is x3 .
96
Example 3.5.14.
È È
x7 y 3 = x6 xy 2 y
È
√
= x6 y 2 xy
√
= x3 y xy.
Again, the idea was to separate out as much of the radicand as possible while leaving the
exponents as integers. We accomplished this by finding the largest perfect square factor of
x7 y 3 we could.
Example 3.5.15.
È √
3
√
3
È
3
27x6 (x + 1)5 = 27 x6 3 (x + 1)5
È
= 3x2 3 (x + 1)3 (x + 1)2
È
= 3x2 (x + 1) 3 (x + 1)2 .
Example 3.5.16.
È √
6
È
6
x2 y 3 = x2 6 y 3
√ √
= 3 x y.
In this example, we ended up with radicals having different indices. This is sort of in bad
taste, as it makes it more difficult to see what we have, rather than less. It isn’t wrong to
have different indices, but neither is it attractive, so we will avoid it when we simplify.
16. (2x)
3
11 6
23. x 5
1
20. x 5 24. t− 3
7
97
Identify the radicand and the index in each radical expression. Then simplify as much as
possible.
√5
√ √4
37. x6 41. 5 45. x11
√ Ê √
3 6 5 46.
3
54x8
38. 64x 42.
27 √
√ √ 47. y 9
39. 25x 4
43. 15x3 Ê
√ √ 1
4
40. x2 44. 75 48.
8
24623451000000000000000
is. Scientific notation is a way of writing very large or very small numbers in a compact and
easy to read form.
Scientific notation relies on our base-10 positional system. Recall that the number 1234.56
means 1 thousand, two hundreds, 3 tens, 4 ones, 5 tenths, and 6 hundredths. The value of
each position is exactly 10 times the value of the position to its immediate right, and the
value of the position directly to the left of the decimal point is one.
98
This makes it very easy to compute 10 (or 100 or 1000 or...) times a given number:
The effect is just to shift the decimal point further to the right. Notice that the number of
places the decimal point is shifted is equal to the exponent on the 10 shown to the far right.
1 1 1
Likewise, it is easy to compute 10 (or 100 or 1000 or ...) of a given number:
1
(1234.56) · = 123.456 (= 1234.56 × 10−1 )
10
1
(1234.56) · = 12.3456 (= 1234.56 × 10−2 )
100
1
(1234.56) · = 1.23456 (= 1234.56 × 10−3 ).
1000
Again, the decimal point is just shifted, this time to the left. Also, the number of places
it is shifted is just the size of the exponent on the 10. (Incidentally, it is called a decimal
point because our system is base 10, corresponding to the Latin prefix deci-.)
Scientific notation takes advantage of these principles.
Definition 3.6.1. A number is said to be expressed in scientific notation if it is of the
form
x × 10n ,
where 1 ≤ x < 10 and n is an integer.
Example 3.6.1. The number
3.367 × 104
is in scientific notation, but
33.67 × 103
is not since 33 is greater than 10. Notice that these two numbers are equal:
Example 3.6.2. The number 0.644×103 is not in scientific notation since 0.644 < 1. However,
6.44 × 102 , which is equal to the original number, is in scientific notation.
Example 3.6.3. We have that 5.37 × 10−5 = 0.0000537. To see this, simply shift the decimal
point 5 places to the left.
99
2.346 × 109 .
Example 3.6.6. The rest mass of a proton, one of the constituent parts of the nucleus of an
atom, is 1.67 × 10−27 kilograms. Very small! The rest mass of an electron is 9.11 × 10−31
1
kilograms, about 1000 that of the proton! On the other hand, the mass of the Earth is about
24
5.98 × 10 kilograms.
None of these quantities is easily expressed in our standard notation, but they are easily
expressed in scientific notation. [Source: Fundamentals of Physics, Third Edition, Halliday
and Resnick, Wiley (1988).
Example 3.6.7. The speed of light in a vacuum is about 3.00 × 108 meters per second, or
300,000,000 meters per second.
100
Distributive Laws
We presented the properties of operations in Chapter 1, and we have reproduced them below
for convenience.
a + (−a) = (−a) + a = 0.
9. Every real number except 0 has a multiplicative inverse: if a is a real number, then
1 1
a· = · a = 1.
a a
101
102
1. (Distributing)
(a) a(b + c) = ab + ac
(b) (b + c)a = ba + ca
2. (Factoring)
(a) ab + ac = a(b + c)
(b) ba + ca = (b + c)a.
Notice that the second form is really saying the same thing as the first form; we have
just used the symmetric property of equality to rewrite it. When we apply the second form,
we refer to the process as factoring.
Why should the distributive property work? Consider the following example.
Example 4.1.1. Applying the order of operations, we see that
4(3 + 9) = 4 · 3 + 4 · 9 = 12 + 36 = 48.
The diagram below illustrates the distributive property for this example.
9+3
{
4
{
{
{
9 3
The idea is that the total number of squares is the same, regardless of whether we count
the first 9 columns separately from the last 3 columns, or whether we count all 12 columns
together. In fact, figures like the one above motivate the distributive law in the first place.
103
Example 4.1.2.
We often use the distributive law to combine like terms in a polynomial expression.
Example 4.1.3. Simplify 4x + 7x.
Solution: Notice that both terms contain a factor of x, so we have an expression of the
form in 2b above. Therefore,
4x + 7x = (4 + 7)x
= 11x.
That is, to add two polynomial expressions, we simply combine their like terms.
Example 4.1.4.
Again, all we have really done is apply the Distributive Law 2b to the x2 , x, and constant
terms. Also, the sign is part of the coefficient, so we wrote the coefficient of x as (5 − 7); we
could just as well have written it as 5 + (−7).
Example 4.1.5.
Notice here that the second polynomial did not have an x4 term, so the coefficient 6 of
the x4 term in the first polynomial was unchanged. Likewise, the coefficient 2 of x2 in the
second polynomial was unchanged.
104
Example 4.1.6.
2 2 3 3 2 2 7 2 3 2 −3 2 7
x − x+2 + x + x− = + x + + x+ 2−
5 4 8 3 4 5 8 4 3 4
2 · 8 + 3 · 5 2 −3 · 3 + 2 · 4 8 7
= x + x+ −
5·8 4·3 4 4
31 1 1
= x2 − x + .
40 12 4
Combining like terms can be very tedious if you have to consciously employ the distribu-
tive law every time. You should practice combining like terms until you are comfortable
computing
3x3 + 7x3 = 10x3
rather than
3x3 + 7x3 = (3 + 7)x3 = 10x3 .
However, by the same token, you should continue writing the middle step until you do
become comfortable skipping it. Whether you write the middle step or not, though, you
should always be aware that combining like terms makes use of the distributive law. In the
rest of this book, we will sometimes write the middle step, and sometimes omit it.
The distributive laws can also be used to factor polynomial expressions.
Example 4.1.7. Factor 3x2 + 5x.
Solution: Since each term has a factor of x, we may apply 2b from above. Thus,
3x2 + 5x = 3x · x + 5x
= (3x + 5)x.
We cannot factor (in a meaningful way) any further since the remaining terms do not
share a common factor. (They are not like terms.)
Example 4.1.8. Apply the distributive law to factor out a common factor from 4x3 −12x2 +8x.
Solution 1: The three terms all share a factor of 4x:
4x3 − 12x2 + 8x = 4x(x2 ) − 4x(3x) + 4x(2)
= 4x(x2 − 3x + 2).
Solution 2: Strictly speaking, the three terms also share a factor of 8x :
1 3
4x − 12x + 8x = 8x x2 − 8x x + 8x(1)
3 2
2 2
1 2 3
= 8x x − x + 1 .
2 2
105
Typically, though, when we factor polynomials with integer coefficients, we also want the
factors to have integer coefficients, so we would most likely prefer solution 1.
Both solutions share have something in common: the factor we looked for was the highest
power of x occurring in the polynomial expression.
Example 4.1.9.
Notice how in all of these examples we are using the Multiplication of Like Bases Theorem
in reverse: when we see that x8 and x3 both have a factor of x3 , we rewrite x8 as x5 x3 . Check
that this is legitimate!
Example 4.1.10.
x2 2x2 1 2
− = · x2 − · x2
3x + 1 x − 2 3x + 1 x−2
1 2
= − x2 .
3x + 1 x − 2
The distributive laws apply to rational expressions as well as to polynomial expressions.
Example 4.1.11.
5 1 4 1 1
x 2 − 2x 2 = x 2 x 2 − 2x 2
4 1
= x2 − 2 x2
1
= (x2 − 2)x 2 .
1
As before, we looked for the highest power of x the two terms shared; in this case, it was x 2 .
The important thing to remember from this is that the distributive law and factoring are
two sides of the same coin; their difference is only a matter of perspective.
Example 4.2.2.
3x2 (−2x5 − 4x2 ) = 3x2 (−2x5 ) − (3x2 )(−4x2 )
= −6x2+5 − (−12x2+2 )
= −6x7 + 12x4 .
Example 4.2.3.
(3 + 2t + 4t2 )t = [(3 + 2t) + 4t2 ]t Order of Operations
= (3 + 2t)t + (4t2 )t Distributive Law 1b
.
= 3t + (2t)t + 4t2 t Distributive Law 1b
= 3t + 2t2 + 4t3 Multiplication of Like Bases
This example illustrates the fact that the distributive property can be applied to more
than just two terms.
Example 4.2.4.
5(−4x3 + x2 − 3x + 4) = 5(−4x3 ) + 5x2 + 5(−3x) + 5(4)
= −20x3 + 5x2 − 15x + 20.
108
−(3 + x) = (−1)(3 + x)
= (−1)(3) + (−1)(x)
= −3 + (−x)
= −3 − x.
That is, −(3 + x) = −3 − x, so the − sign distributes over the addition, as well. The
effect of the − sign was to change the sign of each term in 3 + x.
7x − (3 + x) = 7x + [−(3 + x)]
= 7x + [−3 − x]
= 7x − 3 − x
= 7x − 3 − 1x
= (7 − 1)x − 3
= 6x − 3.
Example 4.2.7.
−(x − 2) = (−1)(x − 2)
= (−1)x + (−1)(−2)
= −x + 2.
Again, the − sign in front of the polynomial changed each sign in the polynomial.
Example 4.2.8.
Once more, the effect of the − sign was to change the sign of every term in the polynomial.
109
Example 4.2.9.
Example 4.2.11.
2. −(a − b) = −a + b
3. a(b − c) = ab − ac
4. (b − c)a = ba − ca
5. ab − ac = a(b − c)
6. ba − ca = (b − c)a
7. (4x3 + 7x − 3) + (9x2 − 5x + 1)
3 5 4
8. x+2 − x+
7 7 5
9. (7x3 − 3x2 + 4) − (2x3 − 3x2 + 4)
7 4 2 1 4 2 2 4
10. − x4 + x2 + − x − x +
9 9 9 3 3 9
6 2
{
{
4
{
{
(3)(6)
(4)(6)
(3)(2)
(4)(2)
{
3+4
{
6+2
The idea is that the total number of squares is the same, regardless of how we count
them.
The same principle applies even when the expressions contain variables.
112
Example 4.3.2.
Again, notice that each term in the first factor was multiplied by every term in the second
factor. This will always occur.
Example 4.3.4.
Example 4.3.5.
Once more, you can see on the second line that every term in the first
factor (2x + 3) is multiplied by every term in the second factor (x2 − 3x + 4). We can take
advantage of this observation to streamline our work.
113
Example 4.3.6.
Notice also that in each example we apply the definition for addition of polynomials to
combine like terms.
Before we go on, let’s record the fact that each term of a factor is multiplied by every
term of the other factor.
Theorem 23. Let a, b, c, and d be real numbers. Then
(a + b)(c + d) = ac + ad + bc + bd.
You may have heard this referred to as “FOIL” in another algebra class; we will not refer
to it that way in this book. You are far better off learning the underlying principles (like
the distributive law) instead of “clever” names that hide the mathematics.
Example 4.3.7. Let’s be a little more general now. Let c and d be any real numbers. In the
product (x + c)(x + d), we know that each term in the x + c factor will be multiplied by each
term in the x + d factor, so we have
(x + c)(x + d) = x · x + xd + cx + cd
= x2 + dx + cx + cd
= x2 + (d + c)x + cd
= x2 + (c + d)x + cd.
That is,
(x + c)(x + d) = x2 + (c + d)x + cd.
Notice that in the product, the coefficient of x is the sum of the two constant terms from
the factors, and the constant term of the product is the product of the constant terms.
What’s nice about this is that it doesn’t matter what c and d are! Because we used the
symbols c and d instead of specific numbers, when we have specific numbers we can just
substitute them for c and d.
Example 4.3.8. Compute (x + 2)(x + 3).
Solution: We have c = 2 and d = 3 from the last example. Thus,
(x + 2)(x + 3) = x2 + (2 + 3)x + 2 · 3
= x2 + 5x + 6.
114
Notice how much less work this was than what we did in Example 4.3.2 above. It’s the
same problem, and it’s still a consequence of the Distributive Law, but the work we did in
Example 4.3.7 made it much easier to carry out this computation. That is part of the value
of introducing symbols like c and d instead of always dealing with specific numbers.
Example 4.3.9.
(x + 4)(x − 9) = x2 + [4 + (−9)]x + 4(−9)
= x2 − 5x − 36.
Notice again that the − sign is part of the constant term in x − 9.
Example 4.3.10. We can also use Theorem 4.3.1 to multiply more complicated-looking bi-
nomials. For example, let’s compute (2x + 3)(3x + 7). We have a = 2x, b = 3, c = 3x, and
d = 7. Thus
Example 4.3.11. What if we have four unknown coefficients, a, b, c, and d? Let’s find out!
This allows us to quickly compute the product of any two binomials, so we will record it
as a theorem.
Theorem 24. Let a, b, c, and d be real numbers. Then
We claimed that this will allow us to compute the product of any two binomials, but we
already had a theorem for some binomials; namely, (x + c)(x + d) = x2 + (c + d)x + cd. These
two should give us the same result for (x + c)(x + d), so let’s check. We have a = 1 and
b = 1, so
which is the same result we found in Theorem 4.3.7. What a relief! Whenever you learn a
more general result, you should compare it to special cases to make sure it agrees.
Let’s use the theorem in an example.
Example 4.3.12. Compute (3x − 4)(2x + 1).
Solution: We have a = 3, b = 2, c = −4, and d = 1. Thus
For convenience, we gather here the main results of this section. These are well worth
memorizing, as that will greatly increase your proficiency. Here’s a study tip: the more prob-
lems you work, the less time you will have to spend actually “memorizing” these properties.
They will embed themselves in your head if you work them enough.
Theorem 25. Let a, b, c, and d be real numbers. Then
1. (a + b)(c + d) = ac + ad + bc + bd
2. (x + c)(x + d) = x2 + (c + d)x + cd
3. (ax + c)(bx + d) = abx2 + (ad + bc)x + cd
1. (2 + 1)(3 + 2) 3. (1 + 4)(3 + 5)
2. (3 + 3)(2 + 4) 4. (2 + 4)(4 + 5)
(x + 3)(x + 3) = x2 + (3 + 3)x + 3 · 3
= x2 + 6x + 9.
117
What makes this form easy to work with is that the coefficient of x is just twice the
constant term of a factor, and the constant term is the square of the constant term of the
factor.
Example 4.4.2. Compute (x + a)2 .
Solution:
(x + a)2 = (x + a)(x + a)
= x2 + (a + a)x + a · a
= x2 + 2ax + a2 .
(x + a)2 = x2 + 2ax + a2 .
(x + 7)2 = x2 + 2 · 7x + 72
= x2 + 14x + 49.
Example 4.4.4.
BEWARE: Students are often tempted to just square each term of the factor and add
those. However, carefully using the distributive law shows us that there is also an x-term
that must not be neglected. Don’t make the mistake of ignoring it!
Example 4.4.5. What if we have (3x + 4)2 ? We can still compute as in the previous section.
Notice that the coefficient of x2 in the product is the square of the coefficient of x in the
factor, or 32 . The coefficient of x in the product is twice the product of the coefficients in
the factor, or 2(3 · 4). The constant term in the product is the square of the constant term
in the factor, 42 = 16.
Example 4.4.6. Compute (a + b)2 .
Solution:
(a + b)2 = (a + b)(a + b)
= (a · a) + (ab + ba) + b · b
= a2 + 2ab + b2 .
(a + b)(a − b) = a2 − b2 .
Example 4.4.12.
Example 4.4.13. The difference of squares form can also make it easier to multiply some
large numbers in your head. For example,
32 · 28 = (30 + 2)(30 − 2)
= 302 − 22
= 900 − 4
= 896.
The trick is to find a number, like 30, that you can square in your head easily, but this
isn’t always possible.
120
Example 4.4.14.
76 · 84 = (80 − 4)(80 + 4)
= 802 − 42
= 6400 − 16
= 6384.
Each expression below is a perfect square trinomial. Determine what binomial was squared
to attain the given trinomial.
Each expression below is a difference of squares. Determine which two binomials were mul-
tiplied to attain the given difference of squares.
1. ac + ad + bc + bd = (a + b)(c + d)
2. x2 + (a + b)x + ab = (x + a)(x + b)
We have just used the fact that equality is symmetric to reverse the equations. Now let’s
see how these help us factor.
Example 4.5.1. Factor x2 − 4.
Solution: We can recognize that this is a difference of squares;
x2 − 4 = x2 − 22 .
x2 − 22 = (x + 2)(x − 2).
(4x)2 − 52 .
The difference of squares form is usually an easy one to recognize. Perhaps the next
simplest is the perfect square form.
Example 4.5.3. Factor x2 + 10x + 25.
Solution: We can rewrite this as x2 + 2 · 5x + 52 , so we have a perfect square. Thus,
x2 + 10x + 25 = (x + 5)2 .
x2 + 12x + 36 = (x + 6)2 .
More general forms can be more difficult to factor, but we can use part 2 of Theorem
4.5.1 to detect a pattern. We have that
Thus, in order to factor such a trinomial, we need to find a factor pair a and b of the constant
term (ab) whose sum is the coefficient of x (a + b). (Study the equation until that is clear
to you. Remember, ab is the product of a and b, and a + b is the sum of a and b.)
Example 4.5.6. Factor x2 − 4x + 3.
Solution: The factor pairs of 3 are 1 and 3 and −1 and −3. The two whose sum is −4
(the coefficient of x) are −1 and −3, so we rewrite x2 − 4x + 3 as
Since 3 has only the four factors shown, it wasn’t too difficult to determine which added
up to −4. Let’s try one in which the constant term has more factors.
123
x2 + 8x + 12 = x2 + (2 + 6)x + (2)(6)
= (x + 2)(x + 6) Theorem 4.5.1, part 2.
x2 − 9 = x2 + (3 + (−3))x + (3)(−3)
= (x + 3)(x + (−3)) Theorem 4.5.1, part 2
= (x + 3)(x − 3).
Thus, our principle applies to difference of squares forms, too! Let’s see how it fairs for
perfect square forms.
Example 4.5.9. Factor x2 + 6x + 9.
Solution 1: We can recognize x2 + 6x + 9 as a perfect square trinomial that factors as
(x + 3)2 .
Solution 2: Suppose we didn’t recognize x2 + 6x + 9 as a perfect square trinomial. We
need a factor pair of 9 whose sum is 6 (the coefficient of x). The factor pairs of 9 are 1 and
9 and 3 and 3. The sum of 3 and 3 is 6, so this is the pair we want. We have
x2 + 6x + 9 = x2 + (3 + 3)x + (3)(3)
= (x + 3)(x + 3) Theorem 4.5.1, part 2.
This last example illustrates an important point: Theorem 4.5.1 does not cover all of the
possibilities! For some polynomials, we will just have to wait for a later course.
Let’s move on to some more sophisticated examples. Consider part 3 of Theorem 4.5.1.
The coefficient of x2 is ac, and the constant term is bd; if we multiply these together, we get
acbd. This new number has factors ad and bc whose sum is the coefficient of x, and this is
the idea we will exploit. We’ll make this more concrete with an example.
Example 4.5.11. Factor 2x2 + 7x + 3.
Solution: Since the coefficient of x2 isn’t 1, we can’t use part 2 of Theorem 4.5.1, so
we will have to use part 3. Unfortunately, part 3 of Theorem 4.5.1 is too difficult to apply
directly, so we will come at it from another direction.
The product of 2 and 3 is 6, so we need factors of 6 that add up to 7. The factors 1 and
6 add up to 7, so we will rewrite our polynomial:
2x2 + 7x + 3 = 2x2 + 6x + x + 3.
Notice how we split the 7x term into two new terms, 6x and x. Now we will factor this new
expression by grouping terms in a convenient way.
4x2 + 4x + 3 = 4x2 − 2x + 6x − 3
= (4x2 − 2x) + (6x − 3)
= 2x(2x − 1) + 3(2x − 1) Distributive Law
= (2x + 3)(2x − 1) Distributive Law.
Solution 2: You may be wondering what happens if you break 4x into 6x − 2x instead
of −2x + 6x. Let’s see:
4x2 + 4x + 3 = 4x2 + 6x − 2x − 3
= (4x2 + 6x) + (−2x − 3)
= 2x(2x + 3) − (2x + 3) Distributive Law
= (2x − 1)(2x + 3) Distributive Law.
We end up with the same factors, just written in the opposite order.
125
Work all of the exercises. Factorization becomes easier as you gain experience, and you
only gain experience through practice!
Lines
4=9−5
a = 256,
Wait a minute! How did we get from a − 4 = 252 to a = 256? Why did we do what we
did? Why did it work? In the next several examples, we will explore the answers to these
questions by trying to solve some linear equations without contexts.
127
128
Example 5.1.2. Suppose we have the equation x − 8 = 4. We want to know the value of x.
Our approach is to perform operations on both sides of the equation so that the end result
is of the form
x = (something with no x’s).
At that stage, the equation itself tells us the value of x that makes it true. Let’s try this for
the equation x − 8 = 4.
Remember that we want our equation to say x =(something with no x’s). The equation
we are given almost looks like this already, but on the left-hand side of the equal sign there
is an extra “−8” that we don’t want. The simplest way to get rid of that −8 is to add 8 to
both sides of the equation:
(x − 8) + 8 = 4 + 8.
The left-hand side is the same as [x + (−8)] + 8, so we may use the fact that addition is
associative to rewrite our equation as
x + [(−8) + 8] = 4 + 8.
Now, we know that −8 + 8 = 0 and 4 + 8 = 12, so let’s make these changes and see what we
have.
x + 0 = 12.
But this is wonderful! Since x + 0 = x, this just becomes x = 12, so the value of x that
solves this equation is x = 12. We can check that to make sure, too:
(12) − 8 = 4,
Ordinarily, we will not write out so many details. But there are a few things to notice in
here: first, the reason we added 8 to both sides of the equation was so that we could turn
the left-hand side into x + 0, which then turned into just x. It would have been perfectly
legitimate to add 10 to both sides, but then the right-hand side would have been x+2 instead
of just x, so we wouldn’t have been any closer to a solution. In other words, we picked just
the right number to add, namely 8, so that the −8 we didn’t want would be eliminated.
The second thing to notice is that we relied on the fact that addition of real numbers
is associative. Without that special property of addition, we could not have solved this
equation so easily, if at all!
Thirdly, notice that we did add the 8 to both sides of the equation. We did make other
changes on just one side, like rewriting x + 0 as x on the left-hand side, but those are really
the same thing. However, adding 8 to a side changes the side, so you have to make sure you
change the other side in exactly the same way.
Finally, notice that we checked our solution at the end. This is important! This is how
we know with complete certainty that we didn’t make any mistakes. It only takes a couple
of seconds, and it can keep you from having an error in your solution.
129
x + 5 + (−5) = −2 + (−5).
Notice that this time we didn’t show the grouping of the x + 5 on the left-hand side. We are
relying on the associativity of addition of real numbers here! Now our equation simplifies to
x + 0 = −7,
so x = −7 is our solution.
Let’s check again: (−7) + 5 = −2, which is what we wanted.
2x + 5−5 = 13−5
2x + 0 = 8
2x = 8.
130
Now the left-hand side is almost what we want, but there is an extra factor of 2. How do
we get rid of it? Let’s multiply both sides by 12 .
1 1
· (2x) = · 8 (5.1)
2 2
On the left-hand side, we need to make use of the fact that multiplication of real numbers
is associative (just as above we used the fact that addition of real numbers is associative.
Then we can rewrite our equation as
1 1
· 2 x = · 8.
2 2
1 1
Since 2
· 2 = 1 and 2
· 8 = 4, we may simplify this to
(1)x = 4.
Just as in the first set of examples, there are things to notice here. Firstly, we relied on
the associativity of multiplication of real numbers. Without, that we would have been stuck
at 2x = 8.
Secondly, we chose to multiply both sides of 2x = 8 by 12 so that we would get just a (1)x
or x on the left-hand side. It would have been perfectly legitimate to multiply both sides by
3, but we would have ended up with 6x = 24 and been no closer to a solution.
Thirdly, we multiplied both sides of the equation by 12 . This is important!
Finally, do be sure to check your answer.
Let’s recap our strategies. When we see an equation of the form x + a = b, the quickest
way to solve is to add −a to both sides of the equation. This turns the left-hand side into
x + a − a = x + 0 = x, which is what we want on that side.
If we have an equation of the form ax = b (with a 6= 0), the quickest way to solve is to
multiply both sides of the equation by a1 . This turns the left-hand side into a1 · ax = 1x = x,
which is again what we want on that side.
In both cases, we choose an operation that will get us closer to just having x on one side.
If the equation is more complicated, like ax+b = c, solve for a multiple of x first. (Above,
that meant solve for the 2x first.) Then the resulting equation will look like dx = e, which
we know how to solve.
NOTE: There is certainly more than one way to solve ax + b = c; above we offered a
suggestion that might keep the equation a little simpler as you solve, but you do not have to
solve for ax first. Generally speaking, experience will help you decide how to begin. Read
our examples below and try to work every problem. With practice, you can solve a lot of
linear equations in your head!
131
2 5
x−3 = Original equation
3 2
2 5
x−3+3 = +3 Add 3 to both sides
3 2
2 5 6
x+0 = + Simplify and find a common denominator
3 2 2
2 11
x = Simplify
3 2
3 2 3 11 3
· x = · Multiply both sides by *
2 3 2 2 2
33
1·x = Simplify
4
33
x = Simplify
4
33
Thus x = 4
. Let’s check:
2 33 66
· −3= −3
3 4 12
66 36
= −
12 12
30
=
12
5·6
=
2·6
5
= ,
2
27 = 0 + 3x Simplify
27 = 3x Simplify
1 1 1
· 27 = · 3x Multiply both sides by
3 3 3
9 = 1·x Simplify
9 = x Simplify
for y.
Solution: This one starts out with the variable we want to solve for in the denominator,
so we will first multiply both sides by y + 2.
3x − 1
· (y + 2) = 4(y + 2)
y+2
3x − 1 = 4y + 8 Simplification and the distributive law
3x − 1 − 8 = 4y + 8 − 8 Add −8 to both sides
3x − 9 = 4y Simplification
1 1 1
(3x − 9) = (4y) Multiply both sides by
4 4 4
3 9
x− = 1y Simplification and the distributive law
4 4
3 9
Thus, y = x − .
4 4
Ultimately, you will not want to have to write every step. When you have worked enough
problems, you will see the patterns that emerge; certain things always happen. For example,
to solve an equation like x + 6 = 8, we have been writing several steps.
133
x+6=8
x+6−6= 8−6
x+0= 8−6
x = 8 − 6.
We have not simplified the right-hand side because we want to make a point. One who
is comfortable with solving linear equations can go immediately from
x + 6 = 8 to x = 8 − 6
without all of the intermediate steps. This is the level of comfort you are working toward.
The more exercises you work, the sooner you will reach this stage!
We conclude this section with a brief discussion of quadratic equations.
Theorem 31 (Zero-Product Theorem). If the product of two numbers is 0, then one of the
numbers is 0. That is, if ab = 0, then a = 0 or b = 0.
Example 5.1.9. This theorem allows us to solve an equation like x2 + x − 6 = 0. We first
draw on Chapter 4 to factor x2 + x − 6. Factors of −6 whose sum is 1 are 3 and −2. Thus
x2 + x − 6 = (x − 2)(x + 3).
We now have two factors whose product is zero, so by the Zero-Product Theorem, one of
the factors must be zero. That is, either x − 2 = 0 or x + 3 = 0. Both of these are linear
equations, so we can solve them using the techniques of this section. We get that x = 2 or
x = −3.
We do not want to go too far into solving equations like the one above, but we do want
you to be aware that we now have the tools to attack such an equation, at least when we
can factor. The end result is that we just have to solve a couple of linear equations.
Example 5.1.10. Solve 2x2 + x − 3 = 0 for x.
Solution: Since 2(−3) = −6, we need factors of −6 whose sum is 1; these are 3 and −2.
Thus
2x2 + x − 3 = 2x2 − 2x + 3x − 3
= 2x(x − 1) + 3(x − 1)
= (2x + 3)(x − 1).
We may now solve (2x + 3)(x − 1) = 0.
By the Zero-Product Theorem, we must have either 2x + 3 = 0 or x − 1 = 0. Therefore,
either x = − 32 or x = 1.
1. x + 3 = 0 9 2 2 4 5
12. x − = 22. x+ =−
10 5 3 3 3
2. x + 15 = 0
13. 2x + 6 = 0 23. 8x + 7 = 3x − 1
3. x − 11 = 0
14. 3x − 8 = 0 24. 4x + 2 = 11x + 5
4. x − 12 = 7
15. −5x + 12 = 0 25. 4 = 12 − x
5. x + 4 = 8
6. x − 3 = −3 16. 4x − 7 = 5
26. 3x + 5 = 7x − 2
7. x + 9 = 7 17. 12x + 8 = −4
27. −x + 8 = 4x − 3
2 5 18. 3x + 5 = 11
8. x − = 4 1 3 7
3 3 28. x− =− x+
√ √ 19. −2x + 4 = 7 5 4 8 4
9. x + 2 5 = 7 5
4 3 6 2 7 5 1
10. x + = 20. 8x + = 29. 2x + = − x
5 7 7 7 12 12 3
4 3 7
11. x + 15 = − 21. x−2 =9 30. 4x − 6 = x + 1
3 8 2
31. Paperback books cost 50 cents each at Peter’s bookstore. If Theresa paid $9.50 for
paperbacks, how many did she buy?
32. If James had 6 more CD’s, he would have three times as many as Ellen. If Ellen has
7 CD’s, how many does James have?
33. Kathy’s stock portfolio has lost 45% of its value. If it is now worth $28,500, how much
was it originally worth?
34. Bananas were on sale for 33 cents per pound. If Kim bought a frozen pizza for $3.95
and her total was 6.26, how many pounds of bananas did Kim buy?
-2
3
-7 -1 0 2 5 8 9 10
Suppose that we have two real numbers a and b such that a < b. If we increase both a
and b by the same amount, the results have the same relationship. For example, we would
have a + 2 < b + 2 and a − 37 < b − 37. In general, we have the following theorem.
Theorem 32. Let a, b, and c be real numbers.
1. If a < b, then a + c < b + c.
2. If a > b, then a + c > b + c.
We can use this principle to solve linear inequalities.
Example 5.2.2. Solve x + 3 < 7 for x.
Solution: We will add −3 to both sides of the inequality.
x+3 <7
x + 3 + (−3) < 7 + (−3)
x+0 <4
x < 4.
Thus, our solution is {x|x < 4}. This is shown on the number line below. The open circle
around the 4 indicates that 4 is not part of the solution; the heavy line indicates the other
values that solve the inequality.
136
-3 -2 -1 0 1 2 3 4 5
Figure 5.2: {x|x < 4}
x+4≥2
x + 4 + (−4) ≥ 2 + (−4)
x + 0 ≥ −2
x ≥ −2.
That is, the values of x that make x + 4 ≥ 2 are those values of x that are at least −2. We
may write the solution as {x|x ≥ −2}. The number line below shows the graph of this. This
time, there is a darkened circle around −2, indicating that −2 is part of the solution.
-3 -2 -1 0 1 2 3 4 5
Figure 5.3: {x|x ≥ −2}
4x − 5 ≥ 7.
From our experience with solving linear equations, we know that isolating the 4x is probably
a good first step, so we will do that by adding 5 to both sides of the inequality.
4x − 5 + 5 ≥ 7 + 5
4x ≥ 12.
1 1
(4x) ≥ (12)
4 4
x ≥ 3.
Thus, our solution is {x|x ≥ 3}, shown below. The darkened circle on 3 indicates that 3 is
part of the solution.
-3 -2 -1 0 1 2 3 4 5
Figure 5.4: {x|x ≥ 3}
The same principle we used for 41 applies to any positive factor. If a < b, and if c is any
positive number, then ac < bc. Think of the factor of c as stretching or shrinking a line
segment of length a and one of length b. The one that was bigger is still bigger.
a b
ac bc
The lengths of the line segments are in proportion ; the constant of proportionality is c.
Thus
bc b
= ,
ac a
bc
so if ab > 1 (which is to say, b > a), then ac is also greater than one (which is to say, bc > ac).
This is summarized in the theorem below.
Theorem 33. Let a and b be real numbers, and let c be a positive real number.
Solution:
3x + 8 > 2
3x + 8 − 8 > 2 − 8 Subtracting 8 from both sides
3x + 0 > −6
3x > −6
1 1 1
(3x) > (−6) Multiplying both sides by
3 3 3
1·x > −2
x > −2.
Thus our solution is {x|x > −2}, shown below. The open circle around −2 indicates that
−2 is not part of the solution.
-3 -2 -1 0 1 2 3 4 5
Figure 5.6: {x|x > −2}
Solution:
3 1 1 4
x+ ≤ x−
5 4 3 3
3 1 1 1 4 1 1
x+ − ≤ x− − Subtracting 4
from both sides
5 4 4 3 3 4
3 1 16 3
x+0 ≤ x− − Finding a common denominator
5 3 12 12
3 1 19
x ≤ x−
5 3 12
1 3 1 1 19 1
− x+ x ≤ − x+ x− Adding − x to both sides
3 5 3 3 12 3
−1 3 19
+ x ≤ 0− Combining like terms
3 5 12
4 19
x ≤ − Adding fractions
15 12
15 4 15 −19 15
· x ≤ Multiplying both sides by 4
4 15 4 12
−3 · 5 · 19
1·x ≤ Multiplying fractions and factoring
24 · 3
57
x ≤ − Simplifying fractions.
16
¦ ©
57 57
Thus, our solution is x|x ≤ − 16 . The closed circle around − 16 indicates that − 57
16
is part
of the solution.
-57
16
-5 -4 -3 -2 -1 0 1 2 3
¦ ©
57
Figure 5.7: x|x ≤ − 16
-b -a 0 a b
Figure 5.8: Comparing −a and −b
That this will always happen is the recorded in the next theorem.
−4x < −8
− 41 (−4x) > − 14 (−8)Multiplying both sides by − 14 NOTE CHANGE
1 · x > 2,
so {x|x > 2} is our solution. Notice that the direction of the inequality changed, as the
Multiplication by a Negative Theorem says it must.
-3 -2 -1 0 1 2 3 4 5
Figure 5.9: {x|x > 2}
make it correct. It will save you from making errors and it will make your solutions much
easier to follow.
Many students also try something like that when solving an equation (when division by a
negative doesn’t change anything) or when dividing by a positive (which leaves an inequality
alone). Don’t do it then, either! It may not be wrong, but it is still sloppy and lazy, and
it will encourage bad habits. The “extra” line it takes to do it carefully is well worth the
effort.
We now return you to your regularly scheduled lesson.
Example 5.2.9. Solve 2x + 7 < 5x + 3.
Solution:
2x + 7 < 5x + 3
2x + 7 − 7 < 5x + 3 − 7 Subtracting 7 from both sides
2x < 5x − 4 Simplifying
−5x + 2x < −5x + 5x − 4 Adding −5x to both sides
−3x < −4 Combining like terms
1 1
− (−3x) > − (−4) Multiplying both sides by − 13 NOTE CHANGE
3 3
4
x > .
3
Again, multiplying
¦
both ©sides by a negative number changed the direction of the inequality.
Our solution is x|x > 43 , shown below.
4
3
-3 -2 -1 0 1 2 3 4 5
¦ ©
4
Figure 5.10: x|x > 3
Example 5.2.10. Let’s conclude this section by solving the problem set out at the beginning.
We restate it here for convenience.
Suppose Phone Company A only charges $15 per month, buy they also have a charge of
3 cents per minute of phone time. Phone company B charges a flat rate of $18 per month.
At what point does Phone Company B become cheaper?
Solution: Let’s let t represent the number of minutes of phone time a customer expects
to use in a month. Then Company A will charge 15 + 0.03t dollars for the month, while
Company B still charges 18 dollars. We want to solve
15 + 0.03t > 18
for t. (That says that Company A’s charge is greater than Company B’s.)
142
15 + 0.03t > 18
−15 + 15 + 0.03t > −15 + 18
0.03t > 3
1 1
(0.03t) > (3)
0.03 0.03
t > 100.
Thus, after 100 minutes (1 hour and 40 minutes), Company B becomes cheaper.
7.
-3 -2 -1 0 1 2 3 4 5 10.
-1 0 1 2 3 4 5 6 7
8.
-3 -2 -1 0 1 2 3 4 5 11.
-1 0 1 2 3 4 5 6 7
9.
-5 -4 -3 -2 -1 0 1 2 3 12.
-3 -2 -1 0 1 2 3 4 5
Solve each linear inequality and graph your solution.
31. A car rental company offers two types of rentals. The first type costs $25 per day plus
40 cents per mile. The second type costs $60 per day for unlimited mileage. After how
many miles is the second type cheaper?
32. A tool rental company charges 20 dollars per hour for a front-end loader. A competitor
offers the same front-end loader for 95 dollars plus 10 dollars per hour. After how many
hours is the second company cheaper?
33. One long-distance plan costs $18 per month plus 7 cents per minute. A second plan
costs $25 per month plus 5 cents per minute. After how many minutes is the second
plan cheaper?
34. Membership in a book club costs $40 per year, and members receive a 10% discount
on books. A competing book club charges a membership of 12$ per year, but members
receive only a 5% discount. How much must one spend before the first book club is
the better deal? [Hint: with a 10% discount, a book normally costing x dollars will
cost 0.9x dollars.]
1. Choose a point O on the line, and label it with a 0 (zero). We will call O the origin .
O
0
2. Now choose any other point and label it with a 1. The distance from this second point
to the origin establishes the size of one unit. Customarily, we choose this point to lie
to the right of the origin.
O
0 1
With these markings, we can lay out all of the integers on the line by marking divisions
of this size. Note that the negatives are on one side of 0, and the positives are on the other
side of 0.
2.75
O
-2 -1 0 1 2 3 4 5
-0.5 1.5
To obtain rational numbers, just divide the unit segments into appropriate lengths, as
shown.
The irrational numbers are harder; we will not deal with the construction of the rest of
the real numbers, but assume that every real number corresponds to exactly one point on
the line, and every point on the line corresponds to exactly one real number. The number
corresponding to a given point is its coordinate.
The distance between two points on the line is the (positive) difference in their coordi-
nates.
Example 5.3.1. The distance from 7 to 4 is 3 units, and so is the distance from 4 to 7.
To describe points in a plane, rather than just on a line, we require two coordinates. We
begin with two number lines; for convenience, we will refer to one line as the x-axis and the
other as the y-axis. It is common, though not necessary, to choose the coordinates on the
two lines so that the units are equal.
O O
-2 -1 0 1 2 3 4 5 -2 -1 0 1 2 3 4 5
x y
We then place these two lines in a plane so that they intersect (a) at right angles and
(b) at the point O, which we will still refer to as the origin. Traditionally, we place the
x-axis horizontally on the page with positive numbers to the right, and the y-axis is placed
vertically on the page with positive numbers going up. For simplicity, we label the axes with
just x and y.
y
5
4
II I
3
1
O
0
-2 -1 1 2 3 4 5 x
-1
III IV
-2
This establishes the coordinate system; notice that the lines divide the plane of the page
into four sections, called quadrants. These are numbered with roman numerals I, II, III,
and IV counterclockwise, as indicated.
Let us now see how we can use the coordinate system to identify points in the plane.
Imagine that through each point on the y-axis there is a horizontal line (parallel to the
x-axis). Likewise, imagine that through each point on the x-axis, there is a vertical line
146
(parallel to the y-axis). Together, these lines form a grid. In the figure below, we have only
shown those lines passing through integer points on each axis, but every point on the y-axis
has a horizontal line through it (even 1.3246 and −π), and every point on the x-axis has a
vertical line through it.
y
5
1
O
-2 -1 0 1 2 3 4 5 x
-1
-2
These grid lines fill the entire plane, so no matter what point in the plane we choose, it
will lie on the intersection of two of these lines. Furthermore, it will lie on only one of these
points of intersection. This means that we can identify the point with numbers the two lines
pass through, as in the figure below.
Example 5.3.3. The point P is shown in the figure.
y
5
4 P
1
O
-2 -1 0 1 2 3 4 5 x
-1
-2
Since P lies on the intersection of the vertical line through 2 and the horizontal line
through 4, we will label P with the ordered pair (2, 4). The 2 is the x-coordinate and the
147
4 is the y-coordinate. Collectively, 2 and 4 are known as the coordinates of the point P .
It is customary to write the x-coordinate first.
In summary, we label the point P with the coordinates (2, 4) because to reach P from
the origin, we go to the right 2 units and up 4 units.
Example 5.3.4. On the other hand, if we are given coordinates like, say, (4, −1), how do we
find the corresponding point? Based on the way we built the coordinate system, the point
(4, −1) will lie on the vertical line passing through the 4 on the x-axis and the horizontal
line passing through the −1 on the y-axis. Thus, we count left 4 units from the origin and
down 1 unit from there.
y
5
1
O
-2 -1 0 1 2 3 4 5 x
-1 (4,-1)
-2
There are two main points to keep in mind. First, the first of the two coordinates gives
the horizontal distance from the origin and the second gives the vertical distance from the
origin. Second, positive coordinates go up or to the right, and negative coordinates go down
or to the left. Other than that, it’s just counting units.
3
Example 5.3.5. Locate the points (−1, 3), (2, 5), (−1, −1), (4, −2) and 4
, − 12 in the plane.
148
y
5
(2,5)
4
(-1,3)
3
1
O
0
-2 -1 1 2 3 4 5 x
-1 (3/4,-1/2)
(-1,-1)
-2
(4,-2)
Notice that (−1, 3) and (−1, −1) have the same x-coordinate and that they line on a
vertical line (parallel to the vertical axis). Also, this is the first time in this text that we
have used the notation a/b for the fraction ab . This is a common notation, but it can lead to
ambiguity if it is not used carefully, so we use is as little as possible.
We have omitted a small but important detail up to now: what do we do with points on
the axes? This is really no problem; we simply label them with one coordinate of 0.
Example 5.3.6. The figure shows the points (3, 0) and (0, −2). For (3, 0), we must simply go
to the right 3 units and up 0 units, or not at all. This means that we stay on the x-axis.
For (0, −2), we go to the right 0 units (or not at all) and down 2 units. (Up −2 units and
down 2 units are the same thing.)
y
5
1
(3,0)
O
0
-2 -1 1 2 3 4 5 x
-1
-2 (0,-2)
y
4.
5
4
3.
3 1.
6.
2
5. 2.
7.
1
8. O 16.
0
-2 -1 1 2 3 4 5 x
-1 14.
9. 15.
-2
11. 12. 13.
10.
1 7
17. (1,3) 21. (6, 6) 25. (−1.5, 1.75) 29. ,
4 4
18. (4, −1) 22. (−2, −2) 26. (0, 8) 30. (4, 2)
19. (3, 2) 23. (−3.5, 1) 27. (0, −3) 31. (3.75, −4)
20. (0.5, −4) 24. (2.25, 0) 28. (1, 2) 32. (−5, −3.75)
y
5
1
O
0
-2 -1 1 2 3 4 5 x
-1
-2
If we lay down horizontal and vertical lines as shown below, we see that we create triangles
that are in proportion.
y
5
2 2
1
1
2 2
O
0
-2 -1 1 2 3 4 5 x
-1
-2
This means that the ratio of the sides of the small triangle and the ratio of the sides of
the large triangle are equal:
1 2
= ,
2 4
which we know is true.
The remarkable fact is that the ratio of the sides will be 12 no matter where along the line
we place our triangle! This means that that quantity reflects something special about the
line. Since it is special, we give it a name. The ratio is called the slope of the line. Thus,
the slope of the line shown above is 12 .
151
Definition 5.4.1. Suppose that (x1 , y1 ) and x2 , y2 are two different points on a line. The
slope of the line is the ratio
y2 − y1
m= .
x2 − x1
Examination of the triangles shows that we can use any two points we want; the slope
will come out the same. Many textbooks refer to the change in the y-values, y2 − y1 , as
the rise since that is how far “up” (the page) the line goes for a certain horizontal change.
Likewise, the change in the x-values, x2 − x1 , is the run; it refers to how far horizontally the
line moves. With this terminology, the slope may be thought of as the “rise over the run.”
y
5
3 (x2 , y2)
2 rise = y2 - y1
(x1 , y1)
1
run = x2 - x1
O
0
-2 -1 1 2 3 4 5 x
-1
-2
Figure 5.24: The slope of a line is the rise over the run
y
5
1
O
0
-2 -1 1 2 3 4 5 x
-1
-2
To determine the rise and the run, we need to locate two points on the line, and any two
points will do. Our computations will be easier if we can find points with integer coordinates.
It appears that the points (−1, 3) and (2, −3) are on the line, so the rise is −3 − 3 = −6 and
the run is 2 − (−1) = 3. Therefore, the slope of the line is −6
3
= −2.
Example 5.4.2. Find the slope of the line through the points (4, 1) and (−2, −3).
We may apply our slope formula:
1 − (−3) 4 2
= = .
4 − (−2) 6 3
Example 5.4.3. Find the slope of the line through the points (2.5, −3.3) and (−1.6, −3.2).
Example 5.4.4. Find the slope of the line through the points (3, 2) and (−4, 2).
2−2 0
= = 0,
3 − (−4) 7
so the slope is 0. The line through these points is horizontal.
153
y 5
3
(-4,2) (3,2)
2
-3 -2 -1 0 1 2 3 4 5 x
-1
-2
-3
If the slope of a line is 34 , then for every 3 steps to the right, the line rises 4 units.
Likewise, if the slope of the line is −43
, then for every 3 steps to the right, the line falls 4
units. That is, when the slope of the line is positive, it rises to the right; when the slope
is negative, it falls to the right. In between positive and negative slopes is zero, and a line
with slope zero neither rises nor falls to the right.
y 5 y 5 y 5
4 4 4
3 3 3
2 2 2
1 1 1
-3 -2 -1 0 1 2 3 4 5 x -3 -2 -1 0 1 2 3 4 5 x -3 -2 -1 0 1 2 3 4 5 x
-1 -1 -1
-2 -2 -2
-3 -3 -3
4
Positive Slope 3
Zero Slope Negative Slope −4
3
7
In addition, a line with a slope of goes up 7 units for every 3 units it moves to the
3
4
right; a line with a slope of only goes up 4 units for every 3 units to the right. That is,
3
for positive slopes, the line with the greater slope is steeper (and rises faster).
−7
On the other hand, a line with a slope of goes down 7 units for every 3 units it moves
3
−4
to the right, and a line with a slope of goes down 5 units for every 3 units to the right.
3
154
That is, for negative slopes, the line with the smaller slope is steeper (and falls faster).
y 5 y 5
m = 7/3 m = 4/3
4 4
3 3
2 2
1 1
m = - 7/3 m = - 4/3
-3 -2 -1 0 1 2 3 4 5 x -3 -2 -1 0 1 2 3 4 5 x
-1 -1
-2 -2
-3 -3
7 4
Slopes 3
and 3
Slopes −7
3
and −4
3
Example 5.4.5. Determine whether the three points (8, 1), (4, 6), and (0, 10) are collinear.
(That is, do they lie on the same line?)
Solution: If they lie on the same line, then the slope from (0, 10) to (4, 6) will be the
same as the slope from (4, 6) to (8, 1) since the slope of the line is the same no matter which
two points on the line we choose.
10 − 6 4
1. From (0, 10) to (4, 6): = = −1.
0−4 −4
6−1 5 5
2. From (4, 6) to (8, 1): = =− .
4−8 −4 4
Since these slopes are different, the points do not lie on the same line.
155
y (0,10)
9
8
7 (4,6)
6
5
4
3
2
1 (8,1)
-7 -6 -5 -4 -3 -2 -1 1 2 3 4 5 6 7 8 9 x
-1
-2
-3
-4
-5
-6
-7
Visually, it is close; that’s why it’s nice to have an analytic description of slope. We can’t
always trust our eyes, especially when the points are far apart.
We know from our study of fractions that if the denominator is zero, we have a problem.
Can this occur? That is, is it possible that x1 and x2 are equal? If they are, then the
denominator, x1 − x2 , will be zero and the slope will be undefined. Yes, it can happen: if a
line is vertical, then the x-values of all points on the line are the same. In this case, we say
that the line has no slope.
y 5
-3 -2 -1 0 1 2 3 4 5 x
-1
-2
-3
NOTE: There is a vast difference between having no slope and having a slope of 0. We
saw above that a line with a slope of 0 is horizontal. Now we see that a line with no slope
is vertical. Understanding the slope formula will help you keep these ideas straight.
156
If a line is not vertical, then no two points on the line will have the same x-value, and
the slope will be defined. This gives us the following theorem.
Theorem 35. The slope of a line is undefined if and only if the line is vertical.
We have discussed above what the slope of a line can tell you about the line. What can
comparing the slopes of two lines tell you?
Theorem 36. Two lines are parallel if and only if they have equal slopes.
This is a reasonable claim: if two lines have equal slopes, then every step to the right will
cause both lines to rise exactly the same amount, so they can never intersect. On the other
hand, if the two lines have different slopes, then one will rise faster than the other and they
will have to meet somewhere, so they can’t be parallel.
Example 5.4.6. Determine whether a line with a slope of −3
4
is parallel to the line shown.
y
5
1
O
0
-2 -1 1 2 3 4 5 x
-1
-2
The line appears to pass through the points (−2, 4) and (2, 1); from this, we can determine
its slope.
4−1 3 −3
= = .
−2 − 2 −4 4
Thus, any other line with slope −3
4
will be parallel to this one.
There is also a very attractive relationship between the slopes of perpendicular lines.
Theorem 37. Two lines, neither of which is vertical, are perpendicular if and only if the
product of their slopes is −1.
This is a more sophisticated theorem; we will simply accept its truth for this course. You
may see why this is true if you take a course in analytic geometry.
157
y 5
4 (1,4)
3
(4,3)
2 (2,2)
-3 -2 -1 0 1 2 3 4 5 x
-1
-2
-3
The lines do look perpendicular, but that isn’t good enough. Perhaps the angle between
them is 89.999 degrees! We will use the theorem to find out for sure.
The line that rises to the right (and thus has positive slope) passes through the points
(2, 2) and (4, 3), so its slope is
2−3 −1 1
= = .
4−6 −2 2
The line that falls to the right (and thus has negative slope) passes through the points (2, 2)
and (1, 4), so its slope is
2−4 −2
= = −2.
2−1 1
Since the product of these is
1
(−2) · = −1,
2
the two lines are perpendicular.
1. -7
158
y y y
9 9 9
8 8 8
7 7 7
6 6 6
5 5 5
4 4 4
3 3 3
2 2 2
1 1 1
-7 -6 -5 -4 -3 -2 -1 1 2 3 4 5 6 7 8 9 x -7 -6 -5 -4 -3 -2 -1 1 2 3 4 5 6 7 8 9 x -7 -6 -5 -4 -3 -2 -1 1 2 3 4 5 6 7 8 9 x
-1 -1 -1
-2 -2 -2
-3 -3 -3
-4 -4 -4
-5 -5 -5
-6 -6 -6
2. -7
5. -7
8. -7
y y y
9 9 9
8 8 8
7 7 7
6 6 6
5 5 5
4 4 4
3 3 3
2 2 2
1 1 1
-7 -6 -5 -4 -3 -2 -1 1 2 3 4 5 6 7 8 9 x -7 -6 -5 -4 -3 -2 -1 1 2 3 4 5 6 7 8 9 x -7 -6 -5 -4 -3 -2 -1 1 2 3 4 5 6 7 8 9 x
-1 -1 -1
-2 -2 -2
-3 -3 -3
-4 -4 -4
-5 -5 -5
-6 -6 -6
3. -7
6. -7
9. -7
y y
9 9
8 8
7 7
6 6
5 5
4 4
3 3
2 2
1 1
-7 -6 -5 -4 -3 -2 -1 1 2 3 4 5 6 7 8 9 x -7 -6 -5 -4 -3 -2 -1 1 2 3 4 5 6 7 8 9 x
-1 -1
-2 -2
-3 -3
-4 -4
-5 -5
-6 -6
4. -7
7. -7
10. -7
13. -7
16. -7
y y
9 9
8 8
7 7
6 6
5 5
4 4
3 3
2 2
1 1
-7 -6 -5 -4 -3 -2 -1 1 2 3 4 5 6 7 8 9 x -7 -6 -5 -4 -3 -2 -1 1 2 3 4 5 6 7 8 9 x
-1 -1
-2 -2
-3 -3
-4 -4
-5 -5
-6 -6
11. -7
14. -7
y y
9 9
8 8
7 7
6 6
5 5
4 4
3 3
2 2
1 1
-7 -6 -5 -4 -3 -2 -1 1 2 3 4 5 6 7 8 9 x -7 -6 -5 -4 -3 -2 -1 1 2 3 4 5 6 7 8 9 x
-1 -1
-2 -2
-3 -3
-4 -4
-5 -5
-6 -6
159
On a set of axes, draw two lines with the given slope m. One line should pass through the
given point P , and the other should not.
Determine the slope m of the line through the given points. If the slope is undefined, say so.
1 2 7 2
16. (2, 1) and (3, −2) 20. (2, 7) and (2, −1) 24. ,
3 7
and ,
3 7
17. (4, 6) and (1, 0) 21. (0, 0) and (5, 3) 25. (1, −12) and (3, 6)
18. (5, 2) and (−3, 2) 22. (0, 0) and (−2, −7) 26. (4, −2) and (4, 12)
2 1 8 9 5 2 4 1
19. ,
3 4
and ,
3 4
23. ,
8 5
and ,
3 6
27. (3, 5) and (8, −2)
Find (a) the slope a of a line parallel to a line with the given slope and (b) the slope b of a
line perpendicular to a line with the given slope m.
28. m = 5 5 32. m = −8 7
30. m = − 34. m = −
2 4
2 4
29. m = 31. m = −1 33. m = 35. m = 0
3 15
In the last section, we observed that the slope of a line gives us a great deal of information
about the line. However, it does not give us all the information about a line. Specifically,
since parallel lines have the same slope, the slope only tells us that our line is one of a large
collection of parallel lines. We need another piece of information in order to pin down just
which line is ours.
160
y 5
-3 -2 -1 0 1 2 3 4 5 x
-1
-2
-3
From the figure above, it is fairly clear that each of the parallel lines intersects the y-axis
exactly once, and no two of them intersect the y-axis in the same point. (Otherwise, they
would also intersect each other at that point, which parallel lines don’t do!) This means that
that intersection point with the y-axis is one additional piece of information that will tell us
exactly which line is ours.
Definition 5.5.1. The point of intersection of a non-vertical line with the y-axis is called
the y-intercept of the line. It is the point on the line having an x-coordinate of 0.
We had to specify “non-vertical” because a vertical line either is the y-axis or is parallel
to the y-axis; either way, “the” y-intercept doesn’t make sense.
Example 5.5.1. The y-intercept of the line shown is (0, 3), or just 3, since the x-coordinate
is automatically 0 for the y-intercept.
y 5
3 (0,3)
-3 -2 -1 0 1 2 3 4 5 x
-1
-2
-3
Now take a line with slope m, and suppose that (x, y) is some generic point on that line.
Also suppose that the y-intercept of the line is the point (0, b). (Remember, the y-intercept
is where the x-coordinate is 0.) Since we have two points on the line, we can compute the
slope, which we already know is m:
y−b y−b
= m, or = m.
x−0 x
If we multiply both sides of this equation by x, we see that
y−b
·x=m·x
x
y − b = mx
y = mx + b.
This is our first equation for a line.
Theorem 38. If (x, y) is a point on the line with slope m and y-intercept (0, b), then x and
y satisfy the equation
y = mx + b.
This equation is called the slope-intercept equation of the line.
It is called the slope-intercept equation of the line because we need to know the slope of
the line and the y-intercept.
3
Example 5.5.2. Write an equation of the line with slope − and y-intercept 5.
4
Solution: We may simply plug into the slope-intercept form to find the equation
3
y = − x + 5.
4
The graph of this line is shown below.
y 5
3
3 x+5
2 y= 4
-3 -2 -1 0 1 2 3 4 5 x
-1
-2
-3
3
Figure 5.35: The line y = − x + 5
4
162
Example 5.5.3. The equation y = − 23 x + 2 is an equation of a line. Find the slope and the
y-intercept of this line.
Solution: Since the line is in slope-intercept form, we may simply read off the slope and
the y-intercept. The slope is − 23 and the y-intercept is 2.
Example 5.5.4. Find an equation of the line with slope 0 and y-intercept −3.
Solution: We have y = (0)x − 3 = −3, or just y = −3.
The previous example shows that a horizontal line with y-intercept c has the form y = c.
That is, no matter what value of x we choose, the y-value of a point on the line is c. Similarly,
if a line is vertical (and therefore has no slope), then all of the x-coordinates of points on
that line are equal, and the line has equation x = c.
y 5 y 5
4 4
3 3
2 2
1 1
-3 -2 -1 0 1 2 3 4 5 x -3 -2 -1 0 1 2 3 4 5 x
-1 -1
-2 -2
-3 -3
Example 5.5.5. Find an equation of the line through the point (−2, 1) that has y-intercept
(0, 3).
Solution: This time, we are not given the slope, but we have two points that we can
use to find the slope m:
1−3 −2
m= = = 1.
−2 − 0 −2
Now we can go to our slope-intercept form and find the equation
y = 1 · x + 3,
or
y = x + 3.
163
What if we are not given the y-intercept? For example, suppose we know that the slope
7
of the line is 12 and that the line passes through the point (3, 5). If (x, y) is another (generic)
point on the line, then the slope of the line is
y−5
.
x−3
7
Since we already know that the slope is , these two must be equal:
12
y−5 7
= .
x−3 12
Let’s multiply both sides by x − 3 to simplify the left-hand side.
y−5 7
· (x − 3) = (x − 3)
x−3 12
7
y − 5 = (x − 3).
12
That is, if (x, y) is another point on the line, then x and y make the equation
7
y−5= (x − 3)
12
true. This is an example of an equation of a line in point-slope form. This is the form we
typically use when we are given the slope of a line and a point on the line.
Theorem 39. If (x, y) is a point on the line with slope m passing through the point (x1 , y1 ),
then x and y satisfy the equation y − y1 = m(x − x1 ). This equation is called a point-slope
equation of the line.
If (x1 , y1 ) is a given point, m is the given slope, and (x, y) is any other point on the line,
then we must have
y − y1
=m
x − x1
by the definition of slope. If we multiply both sides of this by x − x1 , we get
y − y1
(x − x1 ) = m(x − x1 )
x − x1
y − y1 = m(x − x1 ),
which is the point-slope form above.
Example 5.5.6. Find an equation of the line with slope −5 that passes through the point
(3, −2).
Solution: One such equation is
y − (−2) = −5(x − 3)
by our point-slope theorem. We can simplify this to
y + 2 = −5(x − 3).
164
Example 5.5.7. Find an equation of the line with slope − 53 that passes through the point
(−2, −1).
Solution: One such equation is y − (−1) = − 53 (x − (−2)), or y + 1 = − 35 (x + 2). We
can also solve this for y, which is equivalent to rewriting it in slope-intercept form.
5
y + 1 = − (x + 2)
3
5 5
y+1=− x− ·2
3 3
5 10
y+1=− x−
3 3
5 10
y =− x− −1
3 3
5 13
y =− x− .
3 3
Suppose now that we are simply given two points on the line. We have already seen that
this will give us the slope, so we really have (in effect) the slope and a point (and another
point). This should be enough information to find an equation of the line.
Example 5.5.8. Find an equation of the line through the points (3, 5) and (7, −1).
Solution: First, we need the slope:
5 − (−1) 6 3
m= = =− .
3−7 −4 2
Now we need to choose a point to use our point-slope form. Let’s try them both and see
what happens.
3
1. Using (3, 5): y − 5 = − (x − 3).
2
3 3
2. Using (7, −1): y − (−1) = − (x − 7), or y + 1 = − (x − 7).
2 2
Oh, dear. These don’t seem to be the same! How can we decide whether or not they
really are? One way might be to solve both of them for y (and thereby put them both in
slope-intercept form) to see if we get the same thing.
165
3
y − 5 = − (x − 3) First equation
2
3 3
y − 5 = − x + (3) Distributive law
2 2
3 9
y−5 = − x+ Simplification
2 2
3 9
y−5+5 = − x+ +5 Adding 5 to both sides
2 2
3 9 10
y = − x+ + Finding a common denominator
2 2 2
3 19
y = − x+ Addition of fractions
2 2
Now let’s try it for the second equation.
3
y + 1 = − (x − 7) Second equation
2
3 3
y + 1 = − x + (7) Distributive law
2 2
3 21
y+1 = − x+ Simplification
2 2
3 21
y+1−1 = − x+ −1 Add −1 to both sides
2 2
3 21 2
y = − x+ − Simplification and finding a common denominator
2 2 2
3 19
y = − x+ Subtraction of fractions
2 2
How wonderful! Both slope-intercept equations are the same! This means that when we
have two points on the line, we may choose either one to use in our point-slope equation.
This leads us to the two-point form of a line, which is really just a shortcut for what we
did in Example 5.5.8.
Theorem 40. If (x, y) is a point on the line through the points (x1 , y1) and (x2 , y2 ), then x
and y satisfy the equation
y2 − y1
y − y1 = (x − x1 ).
x2 − x1
This equation is called a two-point equation of the line.
Example 5.5.9. Find an equation of the line through the points (5, 2) and (−7, 3).
Solution: We can employ the two-point form here. We get
3−2
y−2= (x − 5),
−7 − 5
166
which simplifies to
1
y−2=− (x − 5).
12
Most of the forms we have considered so far have one minor flaw: if the line has no slope
(i.e., is vertical), we cannot use the form. For example, slope-intercept form y = mx + b
requires that we have m, but if the line is vertical, we don’t. The last form addresses this
problem.
Definition 5.5.2. The general form for an equation of a line is an equation of the form
Ax + By + C = 0.
All of the forms we have seen so far can be rewritten in general form. We can rewrite
so A = −m, B = 1, and C = −b. (Verify this for yourself in the exercises.) We can rewrite
so A = −m, B = 1, and C = mx1 − y1 . The other forms can be rewritten as well. Notice
that for a horizontal line, we have (0)x + (1)y + (−c) = 0, and for a vertical line we have
(1)x + (0)y + (−c) = 0.
Example 5.5.10. Write an equation of the line through (4, −2) and (6, 1) in general form.
Solution: We can use the two-point form to get an equation to start with:
1 − (−2)
y − (−2) = (x − 4),
6−4
or
3
y + 2 = (x − 4).
2
We need to have the right-hand side be zero in order to have our equation in general form,
so we will move everything to the left-hand side.
3
y+2 = (x − 4) Original equation
2
3 3 3 3
− (x − 4) + y + 2 = − (x − 4) + (x − 4) Adding − (x − 4) to both sides
2 2 2 2
3 3
− x + (4) + y + 2 = 0 Distributive law and simplification
2 2
3
− x+6+y+2 = 0 Simplification
2
3
− x+y+8 = 0 Simplification
2
The equation is now in standard form with A = − 32 , B = 1, and C = 8.
167
We will see in the next section that general form is very handy for sketching graphs of
lines.
It may seem confusing at first to have so many options for equations of lines. However,
there are a couple of points working in your favor. For one thing, the names of the forms
say what they are. For example, the point-slope form requires one point and the slope. If
you have a point and the slope, this is the form to use.
Secondly, the forms are based on the slope of the line. Did you notice how every time we
introduced a new form (except for general form), we went back to the definition of the slope
to get started? All of the forms are really just rearrangements of each other!
In summary, we have the following forms of equations of lines. The one you choose will
depend on what information you have about the line.
Example 5.5.11. Find an equation of the line through (5, −1) and (4, 3).
Solution: Since we are given two points, we will use the two-point form. Thus
3 − (−1)
y − (−1) = (x − 5)
4−5
y + 1 = −4(x − 5).
We could rewrite this in any form we choose, but we will stop here.
Example 5.5.12. Find an equation of the line through (4, 2) that has slope −4.
Solution: This time, we have the slope and one point, so we will use the point-slope
form. We have
y − 2 = 4(x − 4).
Example 5.5.13. Find an equation of the line through (−2, 6) that is perpendicular to the
line y = 34 x + 1.
Solution: We have one point, so we need either another point or the slope. We have no
way to find another point, but we do know the slope of a line perpendicular to ours, 43 . [Note:
168
“Perpendicular to” is symmetric just like “equal to” is symmetric: if line a is perpendicular
to line b, then line b is perpendicular to line a.] Thus, the slope of our line must be − 34 , so
we may now use the point-slope form to finish.
4
y − 6 = − (x − (−2))
3
4
y − 6 = − (x + 2).
3
4x + 5y − 2 = 0
5y = −4x + 2
1
y = (−4x + 2)
5
−4 2
y= x+ .
5 5
Thus, the slope of the line is − 45 .
Example 5.5.15. Find the y-coordinate of the point on the line y = − 12 x+4 if its x-coordinate
is 5.
Solution: This is another advantage to the slope-intercept form: if we know the x-
coordinate of the point, we can find the y-coordinate. Whatever y is, it must satisfy
1
y = − (5) + 4,
2
so y = − 23 .
1. y = 4x + 2 3. y = x − 1 5. y = 9
3 1
2. y = − x + 5 4. y = x 6. y = 2x − 4
4 6
y y
9 9
8 8
7 7
6 6
5 5
4 4
3 3
2 2
1 1
-7 -6 -5 -4 -3 -2 -1 1 2 3 4 5 6 7 8 9 x -7 -6 -5 -4 -3 -2 -1 1 2 3 4 5 6 7 8 9 x
-1 -1
-2 -2
-3 -3
-4 -4
-5 -5
-6 -6
-7 -7
7. 10.
y y
9 9
8 8
7 7
6 6
5 5
4 4
3 3
2 2
1 1
-7 -6 -5 -4 -3 -2 -1 1 2 3 4 5 6 7 8 9 x -7 -6 -5 -4 -3 -2 -1 1 2 3 4 5 6 7 8 9 x
-1 -1
-2 -2
-3 -3
-4 -4
-5 -5
-6 -6
-7 -7
8. 11.
y y
9 9
8 8
7 7
6 6
5 5
4 4
3 3
2 2
1 1
-7 -6 -5 -4 -3 -2 -1 1 2 3 4 5 6 7 8 9 x -7 -6 -5 -4 -3 -2 -1 1 2 3 4 5 6 7 8 9 x
-1 -1
-2 -2
-3 -3
-4 -4
-5 -5
-6 -6
-7 -7
9. 12.
15. Slope 45 , through (3, −4). 21. Through (6, 1) and (6, −9).
3
24. Perpendicular to y = 8
and through through (−1, 5).
(4, −4).
25. Perpendicular to 4x + 5y − 3 = 0 and
26. Slope 9 and through (5, −2).
Find the y-coordinate of the point on the given line having the given x-coordinate.
34. y = − 34 x + 11
4
,x = 3. 15
37. y = − 16 x+ 3
16
, x = 8.
In the last section, we took information from graphs of lines to find equations for the lines.
In this section, we will use equations of lines to draw their graphs. One key idea to keep in
mind is that two points determine a line. That is, if you know two points on the line, you
can draw the entire line.
y
9
8
7
6
5
4 up 4
3
(0,2) 2
over 1
1
-7 -6 -5 -4 -3 -2 -1 1 2 3 4 5 6 7 8 9 x
-1
-2
-3
-4
-5
-6
-7
There is an equivalent way to find that second point. If we let x = 1, then we see that
the corresponding y satisfies y = 4(1) + 2 = 6, so the point (1, 6) lies on the graph of the
line. Notice that (1, 6) is the other point marked.
3(0) + 5y − 30 = 0, or 5y − 30 = 0.
5y − 30 = 0
5y − 30 + 30 = 0 + 30
5y = 30
1 1
(5y) = (30)
5 5
y = 6.
Thus, the point (0, 6) is on the graph of the given line. Notice that setting x = 0 gave
us the y-intercept, as it should.
Now we need a second point, so we will set y equal to 0; we have
3x + 5(0) − 30 = 0, or 3x − 30 = 0.
172
Just as the point at which the line crosses the y-axis is called the y-intercept , the point
at which the line crosses the x-axis is called the x-intercept. Its y-coordinate is 0 at that
point. In the example above, the x-intercept of the line was (10, 0) or just 10. Often, the x-
and y-intercepts of a line are very convenient points to use in graphing the line.
Example 5.6.3. Sketch a graph of the line with equation 4x + 6y + 12 = 0.
Solution: To find the y-intercept, we set x = 0 and solve 6y + 12 = 0 for y, giving
y = −2. To find the x-intercept, we set y = 0 and solve 4x + 12 = 0 for x, giving x = −3.
These two points suffice to graph the line.
y
9
8
7
6
5
4
3
2
1
-7 -6 -5 -4 -3 -2 -1 1 2 3 4 5 6 7 8 9 x
-1
-2
-3
-4
-5
-6
-7
2
Example 5.6.4. Sketch the graph of the line with equation y = − x + 3.
5
2
Solution: We are given that the y-intercept is 3 and that the slope is − . Thus, we can
5
plot the point (0, 3), and then move to the right 5 units and down 2 units to find the second
point. After that, we can just draw the line through those two points.
173
y
9
8
7
6
5
4
over 2
3
2
1 down 5
x
-7 -6 -5 -4 -3 -2 -1 1 2 3 4 5 6 7 8 9 -10
-1
-2
-3
-4
-5
-6
-7
2
Figure 5.39: The line y = − x + 3
5
y
9
8
7
6
5
4
3
2
1 x
-7 -6 -5 -4 -3 -2 -1 1 2 3 4 5 6 7 8 9 -10
-1
-2
-3
-4
-5
-6
-7
Example 5.6.6. Sketch the graph of the line with equation y + 2 = 12 (x − 3).
Solution: If we set x = 0, we have y + 2 = 12 (−3), so y = − 23 − 2 = − 72 . If we set x = 3,
we have y + 2 = 0, so y = −2. (We chose x = 3 because that made the right-hand side 0,
which is easy to work with.) Thus, the two points we have are (0, −7/2) and (3, −2).
174
y
9
8
7
6
5
4
3
2
1
-7 -6 -5 -4 -3 -2 -1 1 2 3 4 5 6 7 8 9 x
-1
-2
-3
-4
-5
-6
-7
Functions
Functions are very common and very useful mathematical objects. Scientists in all fields use
them to describe the behavior of things they study, as do professionals in many other fields.
Functions are used to describe the motion of a particle, the size of a population as it changes
over time, the amount of a substance created in a chemical reaction, the amount of seed it
takes to plant fields of different sizes, and so on.
In this Unit, we introduce the idea of a function and explore some of the more common
functions.
6.1 Definitions
Example 6.1.1. Consider the table below. It shows shows how the area of a rectangle of
height 4 changes as its length changes; that is, how the area depends on the length.
Length 2 3 4 5 6 7
Area 8 12 16 20 24 28
Notice how each length corresponds to just one area. This is the hallmark of a function.
Definition 6.1.1. Suppose that we have two sets (collections) of objects. A function is a
rule that assigns exactly one object from the second set to each object from the first set. (It
is possible that an object from the second set is used more than once or not at all.) We say
that the objects in the second set are a function of the objects in the first set.
In Example 6.1.1, we have the area of the rectangle as a function of its length. The “first
set” was the set of possible lengths of the rectangle, and the “second set” was the set of
possible areas.
In this course, we primarily will be interested in collections of numbers rather than just
“objects”, but there are times when it makes sense to consider more generic things.
175
176
Example 6.1.2. Every person in your class has a name and a height. Here is a table that
lists names and heights for a fictional class.
Name Sarah Kim Ken Jill Roy Joe Robert Greg Ellen Wayne
Height (inches) 59 64 73 61 70 66 75 75 65 73
Notice that each person has exactly one height, so height is a function of the person.
That is, our “first set” is made up of students, our “second set” is made up of heights, and
every person is assigned exactly one height. Now let’s present the table in a slightly different
way.
Height (inches) 59 61 64 65 66 68 70 73 75
Name Sarah Jill Kim Ellen Joe Debbie Roy Ken Robert
Wayne Greg
Since some heights are paired with more than one person, the person is not a function of
the height.
It is becoming somewhat unwieldy to keep talking about the “first set” and the “second
set,” so let’s introduce some terminology to make our lives easier.
Definition 6.1.2. In the definition of a function, the “first set” is called the domain of the
function, and the “second set” is called the range of the function. Members of the domain
are also sometimes referred to as input, and members of the range are sometimes referred
to as output.
Example 6.1.3. In Example 6.1.1 of this section, the domain was the set of possible lengths
of the rectangle – which means any positive numbers – and the range was the set of possible
areas – also any positive numbers. Thus, both the domain and range are equal to {x|x > 0}.
Example 6.1.4. In the second example, the domain is made up of the people and the range
is made up of the heights. That is, the domain is {Sarah, Jill, Kim, Ellen, Joe, Debbie, Roy,
Mark, Ken, Robert} and the range is {59, 61, 64, 65, 66, 68, 70, 73, 75}.
One way to look at a function is as a reliable machine. If you put a 5 into the machine
and it gives you back a 12, then when you come back two days later and give it a 5 again, it
will give you a 12 again. In Example 6.1.2 above, the second table does not give a reliable
relationship. Sometimes it will give you Ken for a height of 73 inches, and sometimes it will
give you Wayne!
177
Example 6.1.5. The area of a circle of radius r is πr 2 , where π is the famous number approx-
imately equal to 3.14159. Given a particular radius, like 4 cm, the value of
πr 2 = π(4)2 = 16π
will always be the same, so this is reliable. That is, the area of a circle is a function of its
radius.
Example 6.1.6. Let’s define the function with input x and output ±x. The symbol ± is
pronounced “plus or minus” and means that we have two values. Thus, f (2) = 2 or f (2) =
−2. This can’t be a function because it doesn’t always give us the same result when we give
it 2!
Example 6.1.7. Let D = {1, 2, 3, 4, 5, 6}. The function described by the table below does not
have domain D since it is not defined for x = 6. That is, even though it is a function, it is
not a function with domain D.
input 1 2 3 4 5
output 2 3 −2 6 1
Example 6.1.8. Let D = {1, 2, 3, 4, 5, 6} and R = {1, 2, 3, 4, 5, 6, . . .}. (That is, R is the set of
natural numbers.) Define a function with domain D so that for an input of x, then output
is x + 3. There is nothing inherent in the formula for the function that prevents us from
substituting 7 for x, but we have specified that 7 is not in the domain of the function.
Notice that the range of this function includes a lot of values that the function doesn’t
really need. For example, there is no input in the domain of this function that corresponds
to an output of 1. (It’s true that −2 + 3 = 1, but −2 is not in the domain!)
If a function is given by a formula, the domain is assumed to be the largest set of real
numbers for which the function makes sense unless otherwise indicated. (In the example
above, we specifically said that our domain was just {1, 2, 3, 4, 5, 6}.)
Example 6.1.9. The function in this example takes a real number x as its input and returns
the real number 7x − 1 as its output. Since any real number x can be multiplied by 7, and
1 can be subtracted from that result (no matter what it is!), the domain is the set of all real
numbers.
178
Example 6.1.10. The function in this example will take a real number x as its input, and give
back the real number x+1
x
as its output. However, there is a potential problem: if x = 0, then
x+1
x
is not defined! This means that x = 0 cannot be part of the domain of this function. On
the other hand, if x 6= 0, then x+1x
is a real number, so x = 0 is the only number excluded
from the domain. Thus, the domain is the set of all real numbers except 0.
√
Example 6.1.11. What is the domain of the function that gives an output of x for an input
of x?
√
Solution: Remember that x means the positive number whose square is x. If x is neg-
ative, there are no numbers whose square is x, so x is not allowed to be negative. Therefore,
the domain of this function is {x|x ≥ 0}.
x+1
Example 6.1.12. What is the domain of the function that gives an output of for
x2 +x−6
an input of x?
Solution: Since we have a denominator, we have the possibility that some value of x
will make the denominator 0; such values of x must be excluded. In order to see what those
values are, we first need to factor the denominator (which, luckily, we learned how to do in
Chapter 4!).
x2 + x − 6 = (x + 3)(x − 2),
so the values of x we cannot allow are x = −3 and x = 2. Therefore, the domain is {x|x 6=
2, −3}
In a later course, you may see a more formal definition of function. If you do, compare
it to this one until you can convince yourself that they mean the same thing.
1. Domain: {1, 2, 3, 4, 5}. Range: {1, 2, 3, 4, 5}. The output corresponding to an input of
x is x.
2. Domain: {1, 2, 3, 4, 5}. Range: {1, 2, 3, 4, 5}. The output corresponding to an input of
x is x + 1.
3. Domain: the set of all integers. Range: the set of all integers. The output correspond-
x
ing to an input of x is 2 .
x +1
179
4. Domain: the set of all natural numbers. Range: the set of all integers. The output
corresponding to an input of x is −x.
5. Domain: the set of all natural numbers. Range: the set of all natural numbers. The
output corresponding to an input of x is −x.
8. Domain: the set of all natural numbers. Range: the set of all rational numbers. The
output corresponding to an input of x is x+1
x
9. Domain: the set of all real numbers. Range: the set of all real numbers. The output
corresponding to an input of x is x1 .
10. Domain: the set of all real numbers. Range: the set of all real numbers. The output
corresponding to an input of x is x2x+1
11. Domain: the set of all real numbers. Range: the set of all rational numbers. The
output corresponding to an input of x is x − 2.
12. Domain: the set of all real numbers. Range: the set of all real numbers. The output
corresponding to an input of x is x.
Each function below is described in terms of input and output. For the inputs specified, give
the corresponding output.
6.2 Notation
It is usually pretty inconvenient to make a table to represent a function. For example, we
can’t make a complete table for the function in Example 6.1.1 because there are infinitely
many values! Most of the time we have a formula to describe the function, and we also
give the function a name so we can refer to it easily. “The function that gives the area of
a rectangle of height 4 in terms of its length” is rather a lot to say, and so is “the function
that gives an output of 4x for an input of x.”
When we can, we give functions names that will remind us what the functions are (or
do); for example, we might call “the function that gives the area of a rectangle of height 4
in terms of its length” just A, for area. As long as we stay in the context of that problem,
any time we use the letter A, we mean that function. Once we finish with that problem, we
free up the letter A for a different function.
If we have a generic function or one that doesn’t have a special meaning to us (like
“area”), we will typically use the letter f , for “function.” Notice also that function names
are italicized. If we need more than one such function, we usually add letters alphabetically
from there, so we might call our second function g and our third function h.
Example 6.2.1. The table below represents the function f with domain {1, 2, 3, 4, 5} and
range {1, 2, 3, 4, 5, 6, 7, 8, 9, 10}.
f
1 2
2 3
3 5
4 8
5 10
181
Now we can ask, what is the output from f when the input is 2? (It is 3.) This is still a
little unwieldy, though, so we are going to introduce a further simplification.
Definition 6.2.1. If f is the name of a function and x is a member of the domain of f, then
f (x) represents the output from f when the input is x. The symbol f (x) is pronounced, “f
of x”. We also say that f (x) is “f evaluated at x” or “the value of f at x.”
CAUTION: Even though f (x) looks like multiplication, it is not. You will have to be
alert for context to decide whether multiplication or function evaluation is intended.
This allows us easily to describe f in terms of a formula. For example, if we want f to
assign the output 3x − 1 to the input x, we just have to say, “Define f by f (x) = 3x − 1.”
Then if we want to know f (2), we just substitute 2 for x in the formula. Thus,
f (2) = 3(2) − 1
=6−1
= 5.
Also, we write f (x) to indicate the value of f at x, no matter what x is. This is the
nature of the variable x (or any other variable) – it is just a placeholder until you know what
value you want it to take.
This also means that the name of the variable is irrelevant; I can call it anything I want.
The letter x is customary, but t is also used when the variable represents time, i is often used
if the variable has to be an integer, and so on. The point is, don’t get hung up on whether
we write f (x) or f (t); we are really talking about the same thing.
Example
6.2.2. Define the function f by f (x) = x2 −3x+1. Evaluate f (1), f (−1), f (12), f (0),
2
f 5 , and f (t).
Solution: The notation f (x) = x2 − 3x + 1 tells us that the input x is assigned to the
output x2 −3x+1. That is, whatever x we are given as input, we first square it, then subtract
3 times the input from that, and then add 1.
1.
f (1) = 12 − 3(1) + 1
=1−3+1
= −1.
2.
3.
f (12) = 122 − 3(12) + 1
= 144 − 36 + 1
= 109.
4.
f (0) = 02 − 3(0) + 1
=0−0+1
= 1.
5.
2 2 2 2
f = −3 +1
5 5 5
22 3 · 2
= 2− +1
5 5
4 6
= − +1
25 5
4 6 · 5 25
= − +
25 5 · 5 25
4 − 30 + 25
=
25
−1
= .
25
6. f (t) = t2 − 3t + 1. Notice that this looks just like f (x), except that we have a t instead
of an x.
We must take the instructions literally. No matter what value is assigned to x, we sub-
stitute exactly that value into the formula in place of every x that appears.
Example 6.2.3. Define f by f (x) = 4x + 1. Evaluate f (t), f (t + 1), f (♣), and f (Encyclopedia
Bizzarica).
Solution:
1. f (t) = 4(t) + 1 = 4t + 1. All we did was change x to t.
2.
f (t + 1) = 4(t + 1) + 1
= 4t + 4 + 1
= 4t + 5.
The rule says that no matter what value we have for the input, we first multiply it by
4 and then add 1 to the result.
183
3. f (♣) = 4♣ + 1.
√
Example 6.2.4. Define f by f (x) = x + 2. Find the domain of f and evaluate f (2), f (−1), f (7),
and f (3).
Solution: Since we are not allowed to take the square root of a negative number, we
must have x + 2 ≥ 0, or x ≥ −2. Thus, the domain of f is {x|x ≥ −2}.
√ √
1. f (2) = 2 + 2 = 4 = 2.
√ √
2. f (−1) = −1 + 2 = 1 = 1.
√ √
3. f (7) = 7 + 2 = 9 = 3.
√ √
4. f (3) = 3 + 2 = 5.
This is an example of a radical function since the output of the function is a radical
expression.
Example 6.2.5. Define f by f (x) = 4x+3. Find the domain of f and evaluate f (4), f (−7), f (2),
and f (x + 1).
Solution: The domain of f is the set of all real numbers.
4. f (x + 1) = 4(x + 1) + 3 = 4x + 4 + 3 = 4x + 7.
This function is an example of a linear function. Notice that if we replace f (x) with
y, we have y = 4x + 3, which is the slope-intercept form of an equation of a line.
2. f (t) = t2 − 3t + 1.
3. f (4) = 42 − 3(4) + 1 = 5.
184
4.
f (x − 2) = (x − 2)2 − 3(x − 2) + 1
= x2 − 4x + 4 − 3x + 6 + 1
= x2 − 7x + 11.
x+2
Example 6.2.7. Define f by f (x) = 3x−4 . Find the domain of f and evaluate f (−1), f (2), f (5),
and f (x − 3).
Solution: We are not allowed to have the denominator
¦
equal zero, so we need 3x−4 6= 0.
©
4 4
Solving 3x − 4 = 0 gives x = 3 , so the domain is x|x 6= 3 .
−1 + 2 1
1. f (−1) = =− .
3(−1) − 4 7
2+2
2. f (2) = = 2.
3(2) − 4
5+2 7
3. f (5) = = .
3(5) − 4 11
(x − 3) + 2 x−1 x−1
4. f (x − 3) = = = .
3(x − 3) − 4 3x − 9 − 4 3x − 13
This is an example of a rational function since the output of the function is a rational
expression.
We can use the idea of function evaluation to draw a graph of the function; often, it is
easier to interpret the behavior of a function from its graph than from its formula. The idea
is that the input is the x-coordinate and the output is the y-coordinate. Thus, a point on
the graph of f looks like (x, f (x)).
We have already seen a number of graphs of linear functions (their graphs are just lines!),
although we did not refer to them that way at the time.
Our approach to graphing functions is to compute a few points on the graph and then
join them with a smooth curve. Of course, since we are only choosing a few points, it is
possible that our smooth curve misses some important features of the graph, but we should
be able to obtain fairly accurate representations.
Example 6.2.8. Sketch the graph of f (x) = x2 .
Solution: We first make a table of representative values of the function. When we plot
these points, we will see an outline of the graph of the function, which we can then fill in
with a smooth curve.
185
x −3 −2 −1 0 1 2 3
y 9 4 1 0 1 4 9
y
9
8
7
6
5
4
3
2
1
-7 -6 -5 -4 -3 -2 -1 1 2 3 4 5 6 7 8 9 x
-1
-2
-3
-4
-5
-6
-7
10
7.5
5
2.5
-10 -5 5 10
-2.5
-5
-7.5
-10
√
Example 6.2.9. Sketch the graph of f (x) = x + 1.
Solution: We again make a table of representative values. Since we are choosing the
values of x, we may as well choose some convenient ones. Also, the domain of f is {x|x ≥ −1}.
x −1 0 3 8
y 0 1 2 3
y
9
8
7
6
5
4
3
2
1
-7 -6 -5 -4 -3 -2 -1 1 2 3 4 5 6 7 8 9 x
-1
-2
-3
-4
-5
-6
-7
√
Figure 6.3: Some points on the graph of f (x) = x+1
10
7.5
5
2.5
-10 -5 5 10
-2.5
-5
-7.5
-10
√
Figure 6.4: f (x) = x+1
1
Example 6.2.10. Sketch the graph of f (x) = .
x
Solution: We again make a table of representative values. The domain of f is {x|x 6= 0}.
x −3 −2 −1 − 12 − 13 1
3
1
2
1 2 3
y − 31 − 12 −1 −2 −3 3 2 1 1
2
1
3
y
9
8
7
6
5
4
3
2
1
-7 -6 -5 -4 -3 -2 -1 1 2 3 4 5 6 7 8 9 x
-1
-2
-3
-4
-5
-6
-7
1
Figure 6.5: Some points on the graph of f (x) =
x
10
7.5
5
2.5
-10 -5 5 10
-2.5
-5
-7.5
-10
1
Figure 6.6: f (x) =
x
Since x = 0 is not in the domain of f, the graph can’t actually touch the y-axis. Thus,
it is in two pieces – one to the left of the y-axis, and one to the right of the y-axis.
L(50) = $19.95
L(100) = $19.95
L(299) = $19.95
L(300) = $19.95 + 0.05(300 − 300) = $19.95
L(375) = $19.95 + 0.05(375 − 300) = $19.95 + 18.75 = $38.70,
and so on.
The previous example illustrates the way piecewise functions are written. The curly brace
indicates which lines are part of the function. With each line, there is a description of how
the function behaves for certain values of the variable; above, there were two sets of values
to work with: those less than 300 minutes, and those greater than or equal to 300 minutes.
Here is another famous (and important!) example of a piecewise-defined function.
Example 6.3.2. The absolute value function is defined by
¨
x if x ≥ 0
|x| =
−x if x < 0
There are several things to notice about this function. First, you can see from the
examples (as well as the definition) that if a number is positive (or zero), the absolute value
function does nothing to it. If a number is negative, the absolute value function changes the
sign of the number so that it is positive. In a sense, the absolute value function tells you the
“size” of a number, without regard to sign.
Also, the vertical bars (|) are grouping symbols; if something is between them (like 5 − 8
was in part (k) of the last example), the operations inside must be carried out before other
operations.
√
The function 2
È √ x does exactly the same thing as the absolute value function. For exam-
ple, (−3)2 = 9 = 3, and | − 3| = 3, too. Thus,
√
x2 = |x|.
√
We observed in the section on radical expressions that x2 is not just x, and this confirms
it.
Notice from parts (i) and (j) that the absolute value of a sum is not the sum of the
absolute values. That is, |a + b| need not be the same as |a| + |b|.
Here is a graph of the absolute value function.
191
y
9
8
7
6
5
4
3
2
1 x
-7 -6 -5 -4 -3 -2 -1 1 2 3 4 5 6 7 8 9 -10
-1
-2
-3
-4
-5
-6
-7
Notice that to the right of the y-axis, the graph looks just like the graph of y = x, and
to the left of the y-axis, it looks like the graph of y = −x. (Why is that?)
Then
f (−4) = −2
f (−3) = −2
f (−0.003) = −2
f (0) = 1
f (1) = 1
f (40) = 40
f (165234) = 165234.
In each case, we examine the value of x to see which of the three categories it falls into,
and evaluate f based on that.
192
Example 6.3.4. The cost of mailing a package depends on how heavy the package is. The
function f describes this cost.
8
>
>
$3.95 if x is under 2 pounds
<
$5.25 if x is between 2 and 4 pounds
f (x) = >
> $6.50 if x is between 4 and 6 pounds
:
$1.05x if x is more than 6 pounds.
A package weighing 3.5 pounds would cost f (3.5) = $5.25. A package weighing 4.8 pounds
would cost f (4.8) = $6.5. A package weighing 12 pounds would cost $1.05(12) = $12.60.
7. A rental car company charges a daily rate of $24.95 plus 35 cents per mile beyond 200
miles. (The first 200 miles are “free”.) Express the cost of renting a car for one day
in terms of the number of miles driven. Then determine the costs for driving 50, 150,
250, and 350 miles.
193
8. Find a tax table (available at http://www.irs.ustreas.gov) and find the tax correspond-
ing to your income. (The tax tables are a gigantic piecewise-defined function.)
9. A mail-order book company advertises shipping rates of $5 for up to 6 books, and then
$0.75 for each additional book.
Since we must compute both f (x) and g(x), x must be in the domains of both f and g.
Thus, √
(f + g)(4) = 4 + 4 + 1 = 2 + 5 = 7,
while (f + g)(−3) is undefined since f (−3) is undefined.
Also, √ √
(f − g)(x) = f (x) − g(x) = x − (x + 1) = x − x − 1
and √
(f · g)(x) = f (x)g(x) = x(x + 1).
194
Example 6.4.2. Let f (x) = 2x + 1 and g(x) = x − 2. Then (f · g)(x) = (2x + 1)(x − 2). Also,
(f · g)(−1) = (2(−1) + 1)(−1 − 2) = 3.
You may have noticed that we omitted division from our definition above. This is because
division requires an extra condition.
f
Definition 6.4.2. If f and g are functions, we define the quotient of f and g to be g
,
where
f f (x)
(x) = ,
g g(x)
where x is in the domains of both f and g and g(x) 6= 0.
is {x|x 6= ±2}. The numbers we had to remove from consideration, ±2, were those numbers
that make g equal to zero. We have
f 3+1 4
(3) = 2
= .
g 3 −4 5
√
Example 6.4.4. Let f (x) = x and g(x) = 2x − 4. The domain of f is {x|x ≥ 0} and the
domain of g is the set of all real numbers. The domain of
√
f x
(x) =
g 2x − 4
is {x|x ≥ 0 and x 6= 2}. We must again remove any values that make the denominator equal
to zero.
√
f 4 1
(4) = =
g 2(4) − 4 2
√ √
f 8 2
(8) = = .
g 2(8) − 4 6
195
(f ◦ g)(x) = f (g(x))
= f (x2 )
= 2x2 + 1.
Also,
First, notice that f ◦g and g◦f are different; composition of functions is not commutative.
Also, we again caution you to take function evaluation literally. When we have f (x) = 2x+1,
it means that no matter what x is, we double it and then add 1 to find the corresponding
output from f. To evaluate f (x2 ), then, we double x2 and then add 1: f (x2 ) = 2x2 + 1.
(f ◦ g)(−2) = f (g(−2)
= f ((−2)2 )
= f (4)
= 2(4) + 1
= 9.
Solution:
1
(f ◦ f )(x) = f
x
1
= 1
x
x
= 1·
1
= x,
(f ◦ g)(x) = f (g(x))
= f (1 − x)
√
= 1 − x.
which is not defined. The trouble is that the output from g is the input into f, so the output
from g must be in the domain of f ; in this case, it isn’t. In fact, then domain of f ◦ g is
{x|x ≤ 1}.
(f ◦ g)(0) = f (g(0))
= f (1 − 0)
√
= 1
= 1.
(f ◦ g)(−3) = f (g(−3))
= f (1 − (−3))
√
= 4
= 2.
18. Profit is defined as revenue (income) minus cost (expense). If a certain company’s
revenue function is described by R(x) = −x2 + 18x + 6 and its cost function is C(x) =
4x − 22, find its profit function P (x). (Here x is the number of units produced.)
Chapter 7
Section 1.1
Section 1.2
NOTE: Any choice of variables is acceptable.
1. Let n be the number of petitions James files. His court fees are then 10n dollars.
3. Let n be the number of nights Kim stays in the hotel. Then her hotel bill is (9.90+85)n.
5. Let A represent the area (in square feet) to be painted. To paint this much area, we
A
need 400 gallons. (Multiply this number of gallons by 400 to see how much area it
covers.)
7. Let n be the number of times Carlos uses an ATM. His fees are then 4.95 + 0.5n, and
we are told that this is 8.45 dollars. Solving gives n = 7 times.
9. Let p be the number of hours Pat works, and j the number of hours Juanita works.
We are told that p = j + 4, and p = 2j. Thus, 2j = j + 4, so j = 4.
11. Seven hours is 7 · 60 = 420 minutes. If p is the number of pens Thadd made, then
15 + 45p = 420. Solving gives p = 9 pens.
Section 1.3
1. 8 · 5
3. 12 · (3 + 5) (“The sum of three and twelve” is treated as a single number that must be
multiplied by 12.)
199
200
5. 4(8 − 5)
11. The sum of 5 and the product of 9 and the sum of 8 and 2.
13. 5 · (4 + 2) = 5 · 6 = 30.
21. 7 − 12 = 7 + (−12).
23. x − 5 = x + (−5).
Section 1.4
1. 29 11. 4
3. 4 13. (4 · 3) + 1
5. 10 15. (5 − 3) − 4
9. 81 19. [(4 · 6) ÷ 2] · 11
Section 1.5
Section 2.1
In each problem, a strip is one unit.
1. 3. 5.
201
1 13 1
7. 0.25 = 15. 27.
4 35 100
17. 45%
3 1
9. 1.5 = 19. 32% 29.
2 4
21. 22.2%
3 6
11. 14 31.
8 23. 10
100
5 138 7
13. 25. 33.
3 100 10
Section 2.2
8 2·4 2
1. (a) = = . (b) 8(3) = 24 = 12(2).
12 3·4 3
9 3·3 3
3. (a) = = . (b) 9(4) = 36 = 12(3).
12 3·4 4
−4 −1 · 4 −1
5. (a) = = . (b) −4(2) = −8 = 8(−1).
8 2·4 2
28 14 · 2 14
7. (a) = = . (b) 28(20) = 560 = 40(14).
40 20 · 2 20
−5 5(−1) 5
9. (a) = = . (b) (−5)(8) = −40 = (−8)(5).
−8 8(−1) 8
21 (−7)(−3) −7
11. (a) = = . (b) 21(−5) = −105 = 15(−7).
15 (−5)(−3) −5
25.
27.
202
29.
a a·1 1
31. Since a = a · 1, ab
= a·b
= b
by the Fraction Simplification Theorem.
Section 2.3
2 y 2
1. 9. 3 17. 27.
3 4 3
12 3
2 11. 19. 2 + 3x
3. 5 5 29.
5 2
2·5
4 25 21. 4
5. 13. 7·3 31.
5 21 3x
23. 1
9 2 1 2
7. 15. − 25. 33.
7 13 3 3
Section 2.4
9. 1 24
25. , x 6= 0
x
5
1. 11. x
7 27.
3
3 1 7 1
+ = 13. − 1
4 8 8 2 29. , x 6= −1, −2
x+2
3 (4 + t)(t + 2) + (4 − t)
15. 31. , t 6=
3. 20 (4 − t)(t + 2)
3 1 5 −2, 1
11
+ = 17.
8 4 8 15 (2x − 1)(x + 5) − 3x
33. , x 6=
−3 3(x + 5)
19. −5
40
5. 47 + z
37 35. , z 6= 0
4 2 34 21. 3z
+ = 5
7 5 35 37. 5 inches.
5 345
7. 23. 3
39. 4 16
8 56
Section 2.5
203
4 29. 35 8x + 5
1. 57. , x 6= 0, −1
15 2 x(x + 1)
5 31.
3. 3 21
18 59.
33. 1, x 6= 0 5
7 1
5. 2t 3 61. , x 6= 6
6 35. , t 6= − 6
2t + 3 2
7. 8 −58
5 63.
37. 21
9. 18 8
7 1 20x
11. 39. 65. , x 6= −2
2 3 9(x + 2)
13 4 −1126
13. 41. , x 6= 0 67.
6 3 285
15 9 15
15. 43. 68.
−2 64 14
−16 45. 2 5
17. 70.
21 9 3
47.
16 14
1 72. , x 6= −2
19. 5 3
6 49.
4 38
7 74.
21. 19 11
3 51.
17 3(x + 1) 1
33 76. , x 6= −
23. 5 5(2x + 1) 2
28 54. , x 6= 0, −1
x+1
25. 8 78. 45
(x + 2)
27. 4 56. , x 6= −5 80. 9
(x + 5)
Section 2.6
5 8 19. 19.2 cm.
1. x = 9. t =
2 5
165 21. 5 inches.
24 11. x =
3. x = 14
5 23. 100 feet.
13. 12.6 gallons.
5. x = 32
15. 3.1831 meters. 25. x = 3.3
45
7. z = 17. 735 Newtons. 27. x = 15.
4
Section 2.7
204
(x − 2) x 68x − 23
1. , x 6= 0 17. , x 6= 2 31.
(x + 2) x+1 (x + 3)(5x − 1)
1 5 x−3 2 u+1
3. , x 6= 19. , x 6= − 33. , t, u 6= 0
3 3 3x 5 16(t + 1)
1 2 2 3 −118x − 99
5. 4x − 3, x 6= 21. , x 6= , 35.
2 x+1 3 2 (2x − 9)(11x + 3)
−7 (5x + 24)(21x − 4) 40x − 81
7. , x 6= 0 ,x 23. 37.
6=
4 − 3x (x − 3)(x + 2) 12
8 (4x + 5)(7x − 1) 1
9. No simplification is possi- 0, 39. , x 6=
3 3x + 5 7
ble.
4 4x − 14 x−5
11. , x 6= 0 25. , x 6= 0 41. , x 6= 0
7x − 1 3x 3(x + 5)
Section 3.1
1. 24 9. −(4 · 4 · 4) = −64
x
x
x
x3 x3
23. 4 4 4
= =
43 64
x
x
x
x
x4 x4
27. = =
−2 −2 −2 −2
(−2)4 16
29. No. 31. Yes. 33. No.
205
D stands for Degree, LT stands for Leading Term, LC stands for Leading Coefficient, and
CT stands for Constant Term.
Exercise D LT LC CT
35. 3 4x3 4 0
2
37. 2 4x 4 −1
39. 0 15 15 15
41. 2 x2 1 0
5 4 5 7
43. 4 7x 7 4
45. 1 z 1 5
24x2 9x2 12x2 + 5x + 5
47. 49. 51.
5(x + 1) 2(x + 1)2 15x
53. (a), (c), (e), and (s) are like terms. (b) and (d) are like terms. (f), (i), and (o) are like
terms. (g) and (m) are like terms. (h), (p), and (r) are like terms. (j) and (t) are like
terms. (k) and (q) are like terms. (l) and (n) are like terms.
Section 3.2
Section 3.3
1 3. 34 1
1. 7.
33 1411
5. 4x2
206
5 1 x−8 y6
9. 15. 4−15 = 21. =
43 415 y −6 x8
1 1
11. 4 17. x−2 =
t x2
1 1
13. 7 = 2−7 19. x20 y 5 23. = x−6 y −3
2 x6 y 3
Section 3.4
1. 2 3 17. y 4
3
9.
2
1
3. 3 8 19. x 20
11.
27 3 3
5. 32 2 21. x 2 y 4
13. x
1 4
7. 4 15. 343x9 23. x 24 y 165
Section 3.5
√4
√
5 1
13. 33 23. x3 31. x 2
√
15
15. x7 5
Ê
25. x 4 1 2
2 43 y 3
17. 3
5 33. 7
x+1 27. x 6 x3
√
3
19. x2 2
√ x5
9 29. 11
21. x7 y5
2
35. 2x 2
√ √
37. Radicand: x6 . Index: 5. x 5 x. 43. Radicand: 15x3 . Index: 2. x 15x.
√
39. Radicand: 25x4 . Index: 2. 5x2 . 45. Radicand: x11 . Index: 4. x2 x3 .
4
√ √
41. Radicand: 5. Index: 2. 5. 47. Radicand: y 9 . Index: 2. y 4 y.
√ √
49. 4x4 y 2 3y 53. x7 3 33
57. √
√ x 3 x2
√ x2 + 1 √
51. 3t3 6 55. 59. 10 3 3
4x2
Section 3.6
Section 4.1
Section 4.2
√
1. −x − 3 17. −5x6 − x5 − x3 33. − x2 + 4(1 + 6x)
3. −3x + 5 19. x(x − 1) √
35. (3x2 − 5x + 4) x
5. −x5 + 4x3 21. −x3 (x + 4)
1. (9x3 + 21x2 + 3x + 15)
7. 7x2 − 4x 23. 3x(3x2 + 4x + 7)
3. 22x
9. t − 3 25. −(w + 1)
1 5 1 4 1 3 5. −x6 − 4x3
11. x + x + x 27. −2(x + 1)
4 2 4
6
13. 3x − 7x
5
3
5
1
29. −x 4 x 2 + 1
1
7. 4x3 + 9x2 + 2x − 2
√
15. 4x4 − 28x3 + 12x2 + 8x 31. 3 3 x + 1 9. 5x3
Section 4.3
Draw a diagram like that of Figure 4.2 to illustrate each computation.
208
3 2
17. 32x2 − 20x − 33
1
{ (2)(3)
(1)(3)
(2)(2)
(1)(2)
{2+1
19. 3x2 + 5x − 2
21. x2 − 1
23. x2 − 64
1. 3+2
25. 9x2 − 4
3 5 27. 121x2 − 16
{
1 (1)(3) (1)(5)
29. x2 − a2
(4)(3) (4)(5) 1+4
4
31. x2 + 10x + 25
{
33. x2 + 12x + 36
3. 3+5
1 1
2 35. x2 + x +
5. x + 5x − 36 2 16
7. x2 + 10x + 24 37. 25x2 − 70x + 49
9. x2 − 8x + 12 39. x3 − x2 − 5x + 2
11. t2 − 2t − 8 41. x5 − 2x2 − x − 2
1 3
13. x2 + − 43. 6x4 + 22x3 + 43x2 + 43x + 21
4 8
15. 6x2 − 10x − 4 45. −21x4 − 29x3 − 58x2 + 37x + 33
Section 4.4
Section 4.5
Section 5.1
1. x = −3 12 11
15. x = 27. x =
5 5
3. x = 11 1
17. x = −1
29. x = −
5. x = 4 14
3
19. x = − 31. 19 books.
2
7. x = −2
√ 88 33. About $51818.18.
21. x =
9. x = 5 5 3
35. x = 2 or x = 3.
49 8
11. x + 15 = − 23. x = − 37. x = −9 or x = 7.
3 5
13. x = −3 25. x = 8 39. x = − 31 or x = 21 .
Section 5.2
1.
-1 0 1 2 3 4 5 6 7 5.
-3 -2 -1 0 1 2 3 4 5
7. {x|x > −3}
3.
-1 0 1 2 3 4 5 6 7 11. {x|x ≥ 1}
13. 15.
-1 0 1 2 3 4 5 6 7 -7 -6 -5 -4 -3 -2 -1 0 1
{x|x > −1} {x|x ≤ −5}
210
17. -3 -2 -1 0 1 2 3 4 5 25. -3 -2 -1 0 1 2 3 4 5
7 8
x|x > x|x <
3 7
19. -1 0 1 2 3 4 5 6 7 27. -8 -7 -6 -5 -4 -3 -2 -1 0
11
x|x ≥ {x|x ≤ −7}
5
21. -3 -2 -1 0 1 2 3 4 5 29. -3 -2 -1 0 1 2 3 4 5
3
x|x > − 3
2 x|x ≥ −
4
Section 5.3
y
6
21.
4.
5
3
17.
2
29. 19.
25.
1
23.
O
0 x
-3 -2 -1 1 2 3 4 5 6
-1
-2
27.
-3
31.
-4
Section 5.4
12. -7
m =
6 − 25 ; P = (1, 4)
13. − 26. m is undefined.
5
15. 0. y
9
28. a = 5, b = − 51
8
y
7
9
6
8
7
5 5 2
6
4
30. a = − , b =
5
4
3
2 2 5
1
3
2 -7 -6 -5 -4 -3 -2 -1 1 2 3 4 5 6 7 8 9 x
1
-1
-2 1
-7 -6 -5 -4 -3 -2 -1
-1
1 2 3 4 5 6 7 8 9 x -3 32. a = −8, b =
-2
-3
-4
-5 8
-6
-4
-5
-6
14. -7
m = 7 4
10. -7
m = 1; P = (0, 0) 34. a = − , b =
4 7
Section 5.5
1. m = 4, b = 2 5. m = 0, b = 9 9. y = x + 3
1
3. m = 1, b = −1 7. y = − x − 2 11. y = −4.5
2
212
Section 5.6
Find the x- and y-intercepts of each line.
1. x-intercept: − 15
y y
2
. 9
8
9
8
y-intercept: 5. 7
6
7
6
5 5
4 4
3. x-intercept: 25 . 3
2
3
2
1 1
y-intercept: 2. 13. -7 -6 -5 -4 -3 -2 -1
-1
1 2 3 4 5 6 7 8 9 x 19. -7 -6 -5 -4 -3 -2 -1
-1
1 2 3 4 5 6 7 8 9 x
-2 -2
-3 -3
5. x-intercept: 3. -4
-5
-4
-5
y-intercept: 12 . -6
-7
-6
-7
y
9 y = 2x + 8 x−3=0
8
7
6 y
5 9
4 8
3 7
2 6
1 y
5
9
4
7. -7 -6 -5 -4 -3 -2 -1
-1
-2
1 2 3 4 5 6 7 8 9 x 8
7
3
2
-3 6
1
-4 5
-5
-6
4
3 21. -7 -6 -5 -4 -3 -2 -1
-1
-2
1 2 3 4 5 6 7 8 9 x
-7 2
-3
1
-4
2 15. -7 -6 -5 -4 -3 -2 -1
-1
1 2 3 4 5 6 7 8 9 x -5
-6
2 = x+5 -2
-7
3 -3
-4
-5 5
y -6
-7 y=−
9
8 2
7
6 5
5 y + 1 = (x − 4) y
4
3
2 9
8
7
2
6
1
5
9. -7 -6 -5 -4 -3 -2 -1
-1
1 2 3 4 5 6 7 8 9 x 4
3
-2
2
-3
y 1
-4
-5
-6
9
8
23. -7 -6 -5 -4 -3 -2 -1
-1
1 2 3 4 5 6 7 8 9 x
7 -2
-7
6 -3
5 -4
4x + 5y − 10 = 0 4
3
-5
-6
2 -7
1
y
9
8 17. -7 -6 -5 -4 -3 -2 -1
-1
-2
1 2 3 4 5 6 7 8 9 x
−2x + 4y − 6 = 0
7 -3
6 -4
5 -5
4 -6
3 -7
2
1
11. -7 -6 -5 -4 -3 -2 -1 1 2 3 4 5 6 7 8 9 x 4
-1
-2
y+6=− x
-3
-4
5
-5
-6
-7
2x + 12y − 6 = 0
213
Section 6.1
1. Function.
11. Wrong range: Not all outputs are rational. OR: Wrong domain: restricting the domain
to rational numbers would work.
Section 6.2
5 √ 1
5. {x|x 6= −2, 5} 5. Radical. (x 3 =
3
x5 .) 11. −
12
214
10 10
13. −7
7.5 7.5
5 5
2.5 2.5
√
15. 3 -10 -5
-2.5
5 10 -10 -5
-2.5
5 10
-5 -5
-7.5 -7.5
10
7.5
5
19. 25, 25, 9x2, x2 − 2x + 1. 2.5
-10 -5 5 10
-2.5
-5
√
3 -7.5
21. 0, . 25. -10
2
Section 6.3
1. -10 -5 5 10
-2.5
-5
-7.5
-10
f (x) = |x − 1|
10
7.5
5
2.5
3. -10 -5 5 10
-2.5
-5
-7.5
-10
f (x) = |2x|
215
10
7.5
5
2.5
5. -10 -5 5 10
-2.5
-5
-7.5
-10
f (x) = |x| + 1
7. Let C represent the cost of the rental, and let x represent the number of miles driven.
Then ¨
$24.95 if x ≤ 200
C(x) =
$24.95 + 0.35(x − 200) if x > 200.
C(50) = $24.95, C(150) = $24.95, C(250) = $24.95 + 0.35(250 − 200) = $42.45, and
C(350) = $24.95 + 0.35(350 − 200) = $77.45.
9. A mail-order book company advertises shipping rates of $5 for up to 6 books, and then
$0.75 for each additional book.
Section 6.4
Each function h is described in terms of two other functions f and g. Determine the domain
of h.
1. {x|x 6= −1}
4
3. x|x ≥ −
3
5. {x|x 6= ±1}
1 1 f 5 20
7. (f +g)(x) = 2 + has domain {x|x 6= 1}. (−5) = − . (f −g)(4) = .
x +1 x−1 g 39 51
9. (f · g)(−4) = −198. (f + g)(x) = x2 + 2x − 1 has domain the set of all real numbers.
√ g √
11. (f − g)(18) = 3 2 − 6. (x) = 2 with domain {x|x > 0}.
f
13. (f ◦ g)(−7) = 3.
1
15. (f ◦ g)(15) = √ .
13
216
1 √
17. Let f (x) = and g(x) = x.
x−1
1
(a) (f ◦ g)(x) = √ has domain {x|x ≥ 0 and x 6= 1}.
x−1
Ê
1
(b) (g ◦ f )(x) = has domain {x|x > 1}.
x−1
Ê
1
(c) (f ◦ g)(4) = 1. (g ◦ f )(3) = .
2